Mathway | Популярные задачи

1 Найти точное значение sin(30)
2 Найти точное значение sin(45)
3 Найти точное значение sin(30 град. )
4 Найти точное значение sin(60 град. )
5 Найти точное значение tan(30 град. )
6 Найти точное значение arcsin(-1)
7 Найти точное значение sin(pi/6)
8 Найти точное значение cos(pi/4)
9 Найти точное значение sin(45 град. )
10 Найти точное значение sin(pi/3)
11 Найти точное значение arctan(-1)
12 Найти точное значение cos(45 град. )
13
Найти точное значение
cos(30 град. )
14 Найти точное значение tan(60)
15 Найти точное значение csc(45 град. )
16 Найти точное значение tan(60 град. )
17 Найти точное значение sec(30 град. )
18 Найти точное значение cos(60 град. )
19 Найти точное значение cos(150)
20 Найти точное значение sin(60)
21 Найти точное значение cos(pi/2)
22 Найти точное значение tan(45 град. )
23 Найти точное значение arctan(- квадратный корень 3)
24 Найти точное значение csc(60 град. )
25 Найти точное значение sec(45 град. )
26 Найти точное значение csc(30 град. )
27 Найти точное значение sin(0)
28 Найти точное значение sin(120)
29 Найти точное значение cos(90)
30 Преобразовать из радианов в градусы pi/3
31 Найти точное значение tan(30)
32 Преобразовать из градусов в радианы 45
33 Найти точное значение cos(45)
34 Упростить sin(theta)^2+cos(theta)^2
35 Преобразовать из радианов в градусы pi/6
36 Найти точное значение cot(30 град. )
37 Найти точное значение arccos(-1)
38 Найти точное значение arctan(0)
39 Найти точное значение cot(60 град. )
40 Преобразовать из градусов в радианы 30
41 Преобразовать из радианов в градусы (2pi)/3
42 Найти точное значение sin((5pi)/3)
43 Найти точное значение sin((3pi)/4)
44 Найти точное значение tan(pi/2)
45 Найти точное значение sin(300)
46 Найти точное значение cos(30)
47 Найти точное значение cos(60)
48 Найти точное значение cos(0)
49 Найти точное значение cos(135)
50 Найти точное значение cos((5pi)/3)
51 Найти точное значение cos(210)
52 Найти точное значение sec(60 град. )
53 Найти точное значение sin(300 град. )
54 Преобразовать из градусов в радианы 135
55 Преобразовать из градусов в радианы 150
56 Преобразовать из радианов в градусы (5pi)/6
57 Преобразовать из радианов в градусы (5pi)/3
58 Преобразовать из градусов в радианы 89 град.
59 Преобразовать из градусов в радианы 60
60 Найти точное значение sin(135 град. )
61 Найти точное значение sin(150)
62 Найти точное значение sin(240 град. )
63 Найти точное значение cot(45 град. )
64 Преобразовать из радианов в градусы (5pi)/4
65 Найти точное значение sin(225)
66 Найти точное значение sin(240)
67 Найти точное значение cos(150 град. )
68 Найти точное значение tan(45)
69 Вычислить sin(30 град. )
70 Найти точное значение sec(0)
71 Найти точное значение cos((5pi)/6)
72 Найти точное значение csc(30)
73 Найти точное значение arcsin(( квадратный корень 2)/2)
74 Найти точное значение tan((5pi)/3)
75 Найти точное значение tan(0)
76 Вычислить sin(60 град. )
77 Найти точное значение arctan(-( квадратный корень 3)/3)
78 Преобразовать из радианов в градусы (3pi)/4
79 Найти точное значение sin((7pi)/4)
80 Найти точное значение arcsin(-1/2)
81 Найти точное значение sin((4pi)/3)
82 Найти точное значение csc(45)
83 Упростить arctan( квадратный корень 3)
84 Найти точное значение sin(135)
85 Найти точное значение sin(105)
86 Найти точное значение sin(150 град. )
87 Найти точное значение sin((2pi)/3)
88 Найти точное значение tan((2pi)/3)
89 Преобразовать из радианов в градусы pi/4
90 Найти точное значение sin(pi/2)
91 Найти точное значение sec(45)
92 Найти точное значение cos((5pi)/4)
93 Найти точное значение cos((7pi)/6)
94 Найти точное значение arcsin(0)
95 Найти точное значение sin(120 град. )
96 Найти точное значение tan((7pi)/6)
97 Найти точное значение cos(270)
98 Найти точное значение sin((7pi)/6)
99 Найти точное значение arcsin(-( квадратный корень 2)/2)
100 Преобразовать из градусов в радианы 88 град.

решить уравнение sin 7x/2 sin x/2 + cos 7x/2 cos x/2 = cos23x

Записи с меткой «решить уравнение sin 7x/2 sin x/2 + cos 7x/2 cos x/2 = cos23x»

Пример 1.

а) Решить уравнение cos4x+cos2x=0.

б) Найдите все корни этого уравнения, принадлежащие отрезку [-π; π/3].

Решение.

а) Решаем уравнение cos4x+cos2x=0.

Применим формулу               

Tогда данное уравнение примет вид: 2cos3x⋅cosx=0. Отсюда следует, что либо cos3x=0 либо cosx=0.

  • Если cos3x=0, то 3х=π/2+πn, отсюда х=π/6+πn/3, где nϵZ.
  • Если cosx=0, то х=π/2+πn, где nϵZ.

Заметим, что решения уравнения cosx=0 входят в решения уравнения cos3x=0, поэтому общим решением данного уравнения будут числа x=π/6+πn/3, где nϵZ.

б) Найдём все корни этого уравнения, принадлежащие отрезку [-π; π/3].

Рассмотрим общее решение x=π/6+πn/3, где nϵZ на единичной окружности. Здесь значение πn/3 означает, что нужно брать n раз угол π/3. Отмечаем угол π/6, а затем углы, полученные поворотом угла π/6 на π/3, полученный таким образом угол π/2 опять повернём на π/3, получится угол 5π/6, затем угол 5π/6+ π/3=7π/6, следующий угол

7π/6+ π/3=9π/6=3π/2, и, наконец, 3π/2+ π/3=11π/6. Смотрите рисунок 1.

         

Все отмеченные углы рассмотрим на отрезке [-π; π/3]. Смотрим рисунок 2. Получились числа -5π/6; -π/2; -π/6; π/6.

Ответ: а) π/6+πn/3, где nϵZ; б) -5π/6; -π/2; -π/6; π/6.

Пример 2.

а) Решить уравнение cos4x-sin2x=0.

б) Найдите все корни этого уравнения, принадлежащие отрезку [0; π].

Решение.

а) Применим формулу 1-cos2α=2sin2α; тогда данное уравнение примет вид:

1-2sin22x-sin2x=0; 2sin22x+sin2x-1=0. Сделаем замену: sin2x=t.

Получаем равенство: 2t2+t-1=0.

У нас a-b+c=0, поэтому по методу коэффициентов t1=-1, t2=1/2.

  • При sin2x=-1 получаем 2х=-π/2+2πn, отсюда х=-π/4+πn, где nϵZ.
  • При sin2x=1/2 получаем 2х=π/6+2πn и 2х=5π/6+2πn, где nϵZ.

Тогда х=π/12+πn и х=5π/12+πn, где nϵZ.

Рассмотрим решения 2х=-π/2+2πn, 2х=π/6+2πn и 2х=5π/6+2πn на единичной окружности. Возьмём значения 2х при n=0. Углы -π/2, π/6 и 5π/6 отличаются друг от друга на значение 2π/3. Тогда общим решением будут являться числа

2х=π/6+(2π/3)n, отсюда общим решением данного уравнения будут

значения  х=π/12+(π/3)n, где nϵZ.

б) Найдём все корни этого уравнения, принадлежащие отрезку [0; π]. Для этого в общее решение х=π/12+(π/3)n, где nϵZ будем подставлять такие целые значения nϵZ,

чтобы хϵ[0; π].

Возьмём n=0, тогда х=π/12 ϵ[0; π].

При n=1 получим х= π/12+π/3= π/12+4π/12=5π/12 ϵ[0; π].

При n=2 получим х= π/12+2π/3= π/12+8π/12=9π/12=3π/4 ϵ[0; π].

При n=3 получим х= π/12+π, и это значение не входит в заданный отрезок [0; π].

Ответ: а) π/12+(π/3)n, где nϵZ; б) π/12, 5π/12, 3π/4.

Пример 3.

а) Решить уравнение

б) Найдите все корни этого уравнения, принадлежащие отрезку [π; 3π/2].

Решение.

а) Применим формулу cos(α-β)=cosα∙cosβ+sinα∙sinβ; тогда данное уравнение примет вид:

cos3x=cos23x; cos23x-cos3x=0;  cos3x(cos3x-1)=0;

cos3x=0 или cos3x-1=0.

  • Если cos3x=0, то 3х=π/2+πn, тогда х= π/6+(π/3)n, где nϵZ.
  • Если cos3x-1=0, то cos3x=1, тогда 3х=2πm, тогда х=(2π/3)m, где mϵZ.

Общие решения данного уравнения: х=π/6+(π/3)n, где nϵZ и х=(2π/3)m, где mϵZ.

б) Найдём все корни этого уравнения, принадлежащие отрезку [π; 3π/2].

Мы получили значения 3х=π/2+πn и 3х=2πm. Отметим их на единичной окружности, сделав замену 3х=t. Смотрите рисунок 3.

     

Необходимо выполнение условие хϵ[π; 3π/2]. Отсюда следует, что 3хϵ[3π; 9π/2].

Все отмеченные углы рассмотрим на отрезке [3π; 9π/2]. Смотрим рисунок 4. Получились числа 7π/2; 4π; 9π/2. Так как это значения 3х, то делим каждое из них на 3. Получим: 7π/6; 4π/3; 3π/2.

Ответ: а) π/6+(π/3)n, где nϵZ; (2π/3)m, где mϵZ.

б) 7π/6; 4π/3; 3π/2.

Контрольная работа «Тригонометрические уравнения», 10 класс

Контрольная работа №3

Тема: Тригонометрические уравнения.

Вариант 1.

Решите уравнения:

  1. 2sinx + = 0.

  2. сos (

  3. sinx cosx + 2

…………………………………..

  1. 3

…………………………………………

  1. а) Решите уравнение

 2

б) Укажите корни этого уравнения,

принадлежащие отрезку 

Контрольная работа №3

Тема: Тригонометрические уравнения.

Вариант 2.

Решите уравнения:

  1. 2cos x + = 0.

  2. sin (

  3. 2sinx cosx = 3 …………………………………..

  4. 5

…………………………………………

  1. а) Решите уравнение

 2

б) Укажите корни этого уравнения,

принадлежащие отрезку 

Контрольная работа №3

Тема: Тригонометрические уравнения.

Вариант 1.

Решите уравнения:

  1. 2sinx + = 0.

  2. сos (

  3. sinx cosx + 2

…………………………………..

  1. 3

…………………………………………

  1. а) Решите уравнение

 2

б) Укажите корни этого уравнения,

принадлежащие отрезку 

Контрольная работа №3

Тема: Тригонометрические уравнения.

Вариант 2.

Решите уравнения:

  1. 2cos x + = 0.

  2. sin (

  3. 2sinx cosx = 3 …………………………………..

  4. 5

………………………………………….

  1. а) Решите уравнение

 2

б) Укажите корни этого уравнения,

принадлежащие отрезку 

Контрольная работа №3

Тема: Тригонометрические уравнения.

Вариант 1.

Решите уравнения:

  1. 2sinx + = 0.

  2. сos (

  3. sinx cosx + 2

…………………………………..

  1. 3

…………………………………………

  1. а) Решите уравнение

 2

б) Укажите корни этого уравнения,

принадлежащие отрезку 

Контрольная работа №3

Тема: Тригонометрические уравнения.

Вариант 2.

Решите уравнения:

  1. 2cos x + = 0.

  2. sin (

  3. 2sinx cosx = 3 …………………………………..

  4. 5

…………………………………………….

  1. а) Решите уравнение

 2

б) Укажите корни этого уравнения,

принадлежащие отрезку 

3 6 Решить для? cos (x) = 1/2 7 Решить относительно x sin (x) = — 1/2 8 Преобразование из градусов в радианы 225 9 Решить для? cos (x) = (квадратный корень из 2) / 2 10 Решить относительно x cos (x) = (квадратный корень из 3) / 2 11 Решить относительно x sin (x) = (квадратный корень из 3) / 2 12 График г (x) = 3/4 * корень пятой степени x 13 Найдите центр и радиус х ^ 2 + у ^ 2 = 9 14 Преобразование из градусов в радианы 120 градусов 15 Преобразование из градусов в радианы 180 16 Найдите точное значение коричневый (195) 17 Найдите степень е (х) = 2x ^ 2 (x-1) (x + 2) ^ 3 (x ^ 2 + 1) ^ 2 18 Решить для? тангенс (x) = квадратный корень из 3 19 Решить для? sin (x) = (квадратный корень из 2) / 2 20 Найдите центр и радиус х ^ 2 + у ^ 2 = 25 21 Найдите центр и радиус х ^ 2 + у ^ 2 = 4 22 Решить относительно x 2cos (x) -1 = 0 23 Решить относительно x 6x ^ 2 + 12x + 7 = 0 24 Найдите домен х ^ 2 25 Найдите домен е (х) = х ^ 2 26 Преобразование из градусов в радианы 330 градусов 27 Разверните логарифмическое выражение натуральный логарифм от (x ^ 4 (x-4) ^ 2) / (квадратный корень из x ^ 2 + 1) 28 Упростить ((3x ^ 2) ^ 2y ^ 4) / (3y ^ 2) 29 Упростить (csc (x) детская кроватка (x)) / (sec (x)) 30 Решить для? тангенс (х) = 0 31 Решить относительно x х ^ 4-3x ^ 3-х ^ 2 + 3x = 0 32 Решить относительно x cos (x) = sin (x) 33 Найдите точки пересечения по осям x и y х ^ 2 + у ^ 2 + 6х-6у-46 = 0 34 Решить относительно x квадратный корень из x + 30 = x 35 Упростить детская кроватка (x) коричневый (x) 36 Найдите домен у = х ^ 2 37 Найдите домен квадратный корень из x ^ 2-4 38 Найдите точное значение грех (255) 39 Оценить, основание журнала 27 из 36 40 преобразовать из радианов в градусы 2п 41 Упростить (F (x + h) -Fx) / час 42 Решить для? 2sin (x) ^ 2-3sin (x) + 1 = 0 43 Решить относительно x tan (x) + квадратный корень из 3 = 0 44 Решить относительно x sin (2x) + cos (x) = 0 45 Упростить (1-соз (х)) (1 + соз (х)) 46 Найдите домен х ^ 4 47 Решить для? 2sin (x) + 1 = 0 48 Решить относительно x х ^ 4-4x ^ 3-х ^ 2 + 4x = 0 49 Упростить 9 / (х ^ 2) + 9 / (х ^ 3) 50 Упростить (детская кроватка (x)) / (csc (x)) 51 Упростить 1 / (с ^ (3/5)) 52 Упростить квадратный корень из 9a ^ 3 + квадратный корень из 53 Найдите точное значение желто-коричневый (285) 54 Найдите точное значение cos (255) 55 Преобразовать в логарифмическую форму 12 ^ (x / 6) = 18 56 Разверните логарифмическое выражение (основание 27 из 36) (основание 36 из 49) (основание 49 из 81) 57 Недвижимость х ^ 2 = 12 лет 58 Недвижимость х ^ 2 + у ^ 2 = 25 59 График f (x) = — натуральный логарифм x-1 + 3 60 Найдите значение, используя единичную окружность арксин (-1/2) 61 Найдите домен корень квадратный из 36-4x ^ 2 62 Упростить (корень квадратный из x-5) ^ 2 + 3 63 Решить относительно x х ^ 4-2x ^ 3-х ^ 2 + 2x = 0 64 Решить относительно x у = (5-х) / (7х + 11) 65 Решить относительно x х ^ 5-5x ^ 2 = 0 66 Решить относительно x cos (2x) = (квадратный корень из 2) / 2 67 График г = 3 68 График f (x) = — логарифм по основанию 3 из x-1 + 3 69 Найдите корни (нули) f (x) = 3x ^ 3-12x ^ 2-15x 70 Найдите степень 2x ^ 2 (x-1) (x + 2) ^ 3 (x ^ 2 + 1) ^ 2 71 Решить относительно x квадратный корень из x + 4 + квадратный корень из x-1 = 5 72 Решить для? cos (2x) = — 1/2 73 Решить относительно x логарифм по основанию x 16 = 4 74 Упростить е ^ х 75 Упростить (соз (х)) / (1-грех (х)) + (1-грех (х)) / (соз (х)) 76 Упростить сек (x) sin (x) 77 Упростить кубический корень из 24 кубический корень из 18 78 Найдите домен квадратный корень из 16-x ^ 2 79 Найдите домен квадратный корень из 1-x 80 Найдите домен у = грех (х) 81 Упростить корень квадратный из 25x ^ 2 + 25 82 Определить, нечетно ли, четно или нет е (х) = х ^ 3 83 Найдите домен и диапазон f (x) = квадратный корень из x + 3 84 Недвижимость х ^ 2 = 4г 85 Недвижимость (x ^ 2) / 25 + (y ^ 2) / 9 = 1 86 Найдите точное значение cos (-210) 87 Упростить кубический корень 54x ^ 17 88 Упростить квадратный корень из квадратного корня 256x ^ 4 89 Найдите домен f (x) = 3 / (x ^ 2-2x-15) 90 Найдите домен квадратный корень из 4-x ^ 2 91 Найдите домен квадратный корень из x ^ 2-9 92 Найдите домен е (х) = х ^ 3 93 Решить относительно x е ^ х-6е ^ (- х) -1 = 0 94 Решить относительно x 6 ^ (5x) = 3000 95 Решить относительно x 4cos (x-1) ^ 2 = 0 96 Решить относительно x 3x + 2 = (5x-11) / (8лет) 97 Решить для? грех (2x) = — 1/2 98 Решить относительно x (2x-1) / (x + 2) = 4/5 99 Решить относительно x сек (4x) = 2 100 Решите для n (4n + 8) / (n ^ 2 + n-72) + 8 / (n ^ 2 + n-72) = 1 / (n + 9)

Решение сложных триггерных уравнений | Purplemath

Purplemath

  • Решите sin (
    x /2) = cos ( x /2) в полной общности.

Есть разные способы сделать это, но я думаю, что выберу простой выход. Разделив на косинус, я получу тангенс:

(Почему это деление приемлемо? Ну, я не могу делить на ноль, поэтому это деление допустимо, только если косинус не равен нулю. Но в исходном уравнении синус и косинус равны, и они никогда не равны нулю. то же место.И тангенс никогда не равен единице, где косинус равен нулю. Так что в этом случае не было проблемы с делением на ноль. Но всегда не забывайте проверять себя, чтобы быть уверенным.)

MathHelp.com

Тангенс равен 1 для

x /2 = 45 °, 225 ° в первом периоде.Но это упражнение требует ответа «во всей общности». Очевидно, я не могу перечислить все значения решения, потому что их бесконечно много. Так что мне придется использовать формулу.

Из того, что я знаю о графике касательной, я знаю, что касательная будет равна 1 при 45 ° через каждые 180 °. Эти решения для

tan ( x /2) имеют значения 0 ° + 45 °, 180 ° + 45 °, 360 ° + 45 ° и т. Д. Чтобы дать ответ «в общих чертах», я воспользуюсь формулой: x /2 = (180 × n ) ° + 45 °, для всех целых чисел n

Теперь мне нужно решить для самого x .Умножу на 2:


В радианах приведенное выше решение будет иметь вид

x /2 = π / 4, 5π / 4, 9π / 4 и т. Д .; и общее решение будет x = 2π n + π / 2
  • Solve 3tan
    3 ( x ) — 3tan 2 ( x ) — tan ( x ) + 1 = 0 в общем виде.

Я могу разделить это попарно:

3tan 2 ( x ) [tan ( x ) — 1] — 1 [tan ( x ) — 1] = 0

[tan ( x ) — 1] [3tan 2 ( x ) — 1] = 0

желтовато-коричневый ( x ) = 1 или коричневый 2 ( x ) = 1/3

желтовато-коричневый ( x ) = 1 или коричневый ( x ) = ± 1/ sqrt [3]

Первое уравнение решает в первом периоде как:

Второй решает в первом периоде как:

x = 30 °, 150 °, 210 °, 330 °

Чтобы сделать решение «общим», мне нужно сформулировать приведенные выше решения в виде формулы, чтобы учесть каждый период.

Первое решение на 45 ° больше, чем кратное 180 °, поэтому подойдет (180 n ) ° + 45 °. Второе решение — на 30 ° больше, чем кратное 180 °, и (из-за «плюс / минус») также на 30 ° меньше, чем , то же самое кратное, поэтому (180 n ) ° ± 30 ° покроет эту часть .

x = (180 n ) ° ± 30 °, (180 n ) ° + 45 ° для всех целых чисел n


  • Решить на [0, 2π)

Что за черт…?!?

Когда похоже, что ничего не работает, иногда помогает выразить все в терминах синуса и косинуса. Этот процесс, примененный к этому уравнению, дает мне:

Это не намного лучше … но первые два члена имеют общий множитель 2. Если я конвертирую последний член в общий знаменатель с третьим членом, что это даст мне?

Если я разложу на множители 2 из первых двух членов и квадратный корень из 3 и косинус из вторых двух членов, я получу:

Теперь я могу выделить общий множитель:

Уф! Это действительно сработало! Хорошо, теперь мне нужно решить факторы.Первый множитель решает как:

Это уравнение верно при x = 60 ° и, в силу симметрии касательной кривой, также при x = 180 ° + 60 ° = 240 °. В радианах это

.

Второй множитель решает как:

Косинус принимает это значение при x = 30 ° и, в силу симметрии кривой косинуса, также при x = 360 ° — 30 ° = 330 °.В радианах это

. Итак, мое решение:
  • Solve ln (2 — sin
    2 ( x )) = 0 при 0 ° < x <360 °

Натуральный логарифм (ну, любой логарифм ) равен нулю, когда аргумент равен 1, поэтому это дает мне:

2 — грех 2 ( x ) = 1

1 — грех 2 ( x ) = 0

(1 — грех ( x )) (1 + sin ( x )) = 0

1 = грех ( x ) или 1 = –sin ( x )

Из того, что я знаю о синусоиде, мое решение:


  • Журнал решения
    3 (2sin ( x )) = 1/2 на [0, 2π)

Эквивалентное экспоненциальное уравнение по природе логарифмов:

Синус принимает это значение в

, а также в.Тогда мое решение:

Ожидайте, что потребуется фактор (особенно квадратичный) для решения некоторых тригонометрических уравнений, а также ожидайте, что потребуется использовать триггерные тождества. Не бойтесь пробовать разные методы; иногда ваш первый импульс не приводит ни к чему полезному, но ваше второе предположение может сработать. И обратите особое внимание на любые необычно сложные примеры в вашем учебнике, так как они могут содержать подсказки о том, какие уловки вам понадобятся, особенно в следующем тесте.


Вы можете использовать виджет Mathway ниже, чтобы попрактиковаться в решении тригонометрических уравнений. Попробуйте выполнить указанное упражнение или введите свое собственное. (Если вам не сказали решать «в общих чертах», не забудьте указать интервал, как показано ниже.) Затем нажмите кнопку и, для получения наилучших результатов, выберите «Решить через интервал», чтобы сравнить свой ответ с ответом Mathway.

Примечание. Решатель может предоставить только «точные» решения, а иногда и любое решение, если вы в радианах.Используйте дипломы на свой страх и риск!

(Нажмите «Нажмите, чтобы просмотреть шаги», чтобы перейти непосредственно на сайт Mathway для платного обновления.)



URL: https://www.purplemath.com/modules/solvtrig2.htm

7.5 Решение тригонометрических уравнений — предварительное вычисление

Цели обучения

В этом разделе вы:

  • Решите линейные тригонометрические уравнения с синусом и косинусом.
  • Решите уравнения, содержащие одну тригонометрическую функцию.
  • Решите тригонометрические уравнения с помощью калькулятора.
  • Решите тригонометрические уравнения квадратичной формы.
  • Решайте тригонометрические уравнения, используя фундаментальные тождества.
  • Решите тригонометрические уравнения с несколькими углами.
  • Решите задачи прямоугольного треугольника.

Рисунок 1 Египетские пирамиды, стоящие возле современного города. (кредит: Ойсин Малвихилл)

Фалес Милетский (около 625–547 гг. до н.э.) известен как основатель геометрии.Легенда гласит, что он рассчитал высоту Великой пирамиды в Гизе в Египте, используя теорию подобных треугольников , которую он разработал, измерив тень своего посоха. Эта теория, основанная на пропорциях, имеет приложения в ряде областей, включая фрактальную геометрию, инженерию и архитектуру. Часто угол возвышения и угол депрессии находят с помощью одинаковых треугольников.

В предыдущих разделах этой главы мы рассматривали тригонометрические тождества.Тождества верны для всех значений в домене переменной. В этом разделе мы начинаем изучение тригонометрических уравнений для изучения реальных сценариев, таких как определение размеров пирамид.

Решение линейных тригонометрических уравнений с синусом и косинусом

Тригонометрические уравнения, как следует из названия, включают в себя тригонометрические функции. Во многом аналогично решению полиномиальных или рациональных уравнений, только определенные значения переменной будут решениями, если решения вообще есть.Часто мы решаем тригонометрическое уравнение на заданном интервале. Однако так же часто нас просят найти все возможные решения, и, поскольку тригонометрические функции являются периодическими, решения повторяются в течение каждого периода. Другими словами, тригонометрические уравнения могут иметь бесконечное количество решений. Кроме того, как и в случае с рациональными уравнениями, область определения функции должна быть рассмотрена, прежде чем мы предполагаем, что какое-либо решение является действительным. Период синусоидальной функции и косинусной функции равен 2π.2π. Другими словами, каждые 2π2π единиц повторяются значения y- . Если нам нужно найти все возможные решения, мы должны добавить 2πk, 2πk, где kk — целое число, к начальному решению. Вспомните правило, которое дает формат для определения всех возможных решений для функции с периодом 2π: 2π:

sinθ = sin (θ ± 2kπ) sinθ = sin (θ ± 2kπ)

Существуют аналогичные правила для указания всех возможных решений для других тригонометрических функций. Решение тригонометрических уравнений требует тех же методов, что и решение алгебраических уравнений.Мы читаем уравнение слева направо по горизонтали, как предложение. Мы ищем известные закономерности, множители, находим общие знаменатели и заменяем определенные выражения на переменные, чтобы упростить процесс решения. Однако с тригонометрическими уравнениями у нас также есть преимущество использования тождеств, которые мы разработали в предыдущих разделах.

Пример 1

Решение линейного тригонометрического уравнения с использованием функции косинуса

Найдите все возможные точные решения уравнения cosθ = 12.cosθ = 12.

Решение

Из единичного круга мы знаем, что

cosθ = 12 θ = π3,5π3cosθ = 12 θ = π3,5π3

Это решения в интервале [0,2π]. [0,2π]. Все возможные решения дает

π3 ± 2kπ и 5π3 ± 2kππ3 ± 2kπ и 5π3 ± 2kπ

, где kk — целое число.

Пример 2

Решение линейного уравнения с использованием функции синуса

Найдите все возможные точные решения уравнения sint = 12.sint = 12.

Решение

Решение для всех возможных значений t означает, что решения включают углы, превышающие период 2π.2π. Из рисунка 2 видно, что решениями являются π6π6 и 5π6,5π6. Но проблема в том, чтобы указать все возможные значения, которые решают уравнение. Следовательно, ответ

π6 ± 2πk и 5π6 ± 2πkπ6 ± 2πk и 5π6 ± 2πk

, где kk — целое число.

Как добраться

Для данного тригонометрического уравнения решите с помощью алгебры .

  1. Найдите шаблон, который предлагает алгебраическое свойство, такое как разность квадратов или возможность разложения на множители.
  2. Замените тригонометрическое выражение одной переменной, например xx или u.u.
  3. Решите уравнение так же, как и алгебраическое уравнение.
  4. Подставьте тригонометрическое выражение обратно вместо переменной в результирующих выражениях.
  5. Найдите угол.

Пример 3

Решите тригонометрическое уравнение в линейной форме

Точно решите уравнение: 2cosθ − 3 = −5,0≤θ <2π.2cosθ − 3 = −5,0≤θ <2π.

Решение

Используйте алгебраические методы для решения уравнения.

2cosθ − 3 = −5 2cosθ = −2 cosθ = −1 θ = π2cosθ − 3 = −5 2cosθ = −2 cosθ = −1 θ = π

Попробуй # 1

Решите в точности следующее линейное уравнение на интервале [0,2π): 2sinx + 1 = 0. [0,2π): 2sinx + 1 = 0.

Решение уравнений, содержащих одну тригонометрическую функцию

Когда нам задают уравнения, которые включают только одну из шести тригонометрических функций, их решения требуют использования алгебраических методов и единичного круга (см. Рисунок 2).Когда уравнение включает тригонометрические функции, отличные от синуса и косинуса, необходимо учитывать несколько факторов. Проблемы, связанные с величинами, обратными первичным тригонометрическим функциям, необходимо рассматривать с алгебраической точки зрения. Другими словами, мы напишем обратную функцию и найдем углы, используя эту функцию. Кроме того, уравнение, включающее функцию тангенса, немного отличается от уравнения, содержащего функцию синуса или косинуса. Во-первых, как мы знаем, период касательной равен π, π, а не 2π.2π. Кроме того, область касательной — это все действительные числа, за исключением нечетных целых кратных π2, π2, если, конечно, проблема не накладывает свои собственные ограничения на область.

Пример 4

Решение задачи, связанной с одной тригонометрической функцией

Точно решите задачу: 2sin2θ − 1 = 0,0≤θ <2π. 2sin2θ − 1 = 0,0≤θ <2π.

Решение

Поскольку эту проблему нелегко разложить на множители, мы решим, используя свойство квадратного корня. Во-первых, мы используем алгебру, чтобы выделить sinθ.sinθ. Потом найдем углы.

2sin2θ − 1 = 0 2sin2θ = 1 sin2θ = 12 sin2θ = ± 12 sinθ = ± 12 = ± 22 θ = π4,3π4,5π4,7π42sin2θ − 1 = 0 2sin2θ = 1 sin2θ = 12 sin2θ = ± 12 sinθ = ± 12 = ± 22 θ = π4,3π4,5π4,7π4

Пример 5

Решение тригонометрического уравнения с использованием косеканса

Точно решите следующее уравнение: cscθ = −2,0≤θ <4π.cscθ = −2,0≤θ <4π.

Решение

Нам нужны все значения θθ, для которых cscθ = −2cscθ = −2 в интервале 0≤θ <4π.0≤θ <4π.

cscθ = −21sinθ = −2sinθ = −12 θ = 7π6,11π6,19π6,23π6cscθ = −21sinθ = −2sinθ = −12 θ = 7π6,11π6,19π6,23π6
Анализ

Поскольку sinθ = −12, sinθ = −12, обратите внимание, что все четыре решения находятся в третьем и четвертом квадрантах.

Пример 6

Решение уравнения с касательной

Точно решите уравнение: tan (θ − π2) = 1,0≤θ <2π.tan (θ − π2) = 1,0≤θ <2π.

Решение

Напомним, что касательная функция имеет период π.π. На интервале [0, π), [0, π) и под углом π4, π4 касательная имеет значение 1. Однако нам нужен угол (θ − π2). (Θ − π2) . Таким образом, если tan (π4) = 1, tan (π4) = 1, то

θ − π2 = π4θ = 3π4 ± kπθ − π2 = π4θ = 3π4 ± kπ

На интервале [0,2π), [0,2π) имеем два решения:

3π4 и 3π4 + π = 7π43π4 и 3π4 + π = 7π4

Попробуй # 2

Найдите все решения для tanx = 3.tanx = 3.

Пример 7

Определите все решения уравнения с касательной

Определите все точные решения уравнения 2 (tanx + 3) = 5 + tanx, 0≤x <2π.2 (tanx + 3) = 5 + tanx, 0≤x <2π.

Решение

Мы можем решить это уравнение, используя только алгебру. Выделите выражение tanxtanx слева от знака равенства.

2 (tanx) +2 (3) = 5 + tanx2tanx + 6 = 5 + tanx2tanx − tanx = 5−6tanx = −12 (tanx) +2 (3) = 5 + tanx2tanx + 6 = 5 + tanx2tanx − tanx = 5 −6tanx = −1

На единичной окружности есть два угла, значение касательной которых равно −1: θ = 3π4−1: θ = 3π4 и θ = 7π4.θ = 7π4.

Решение тригонометрических уравнений с помощью калькулятора

Не все функции могут быть решены точно с использованием только единичной окружности.Когда мы должны решить уравнение с углом, отличным от одного из специальных углов, нам понадобится калькулятор. Убедитесь, что он установлен на правильный режим, градусы или радианы, в зависимости от критериев данной проблемы.

Пример 8

Использование калькулятора для решения тригонометрического уравнения, содержащего синус

Воспользуйтесь калькулятором, чтобы решить уравнение sinθ = 0,8, sinθ = 0,8, где θθ выражается в радианах.

Решение

Убедитесь, что режим установлен на радианы.Чтобы найти θ, θ, используйте функцию обратного синуса. На большинстве калькуляторов вам нужно будет нажать кнопку 2 ND , а затем кнопку SIN, чтобы вызвать функцию sin − 1sin − 1. На экране отображается sin − 1 (.sin − 1 (. Калькулятор готов к вводу в скобках. Для этой задачи мы вводим sin − 1 (0,8), sin − 1 (0,8) и нажимаем ENTER. Таким образом, с четырьмя десятичными знаками,

sin − 1 (0,8) ≈0,9273 sin − 1 (0,8) ≈0,9273

Решение

Угол измерения в градусах:

. θ≈53.1∘θ≈180∘ − 53,1∘ ≈126,9∘θ≈53,1∘θ≈180∘ − 53,1∘ ≈126,9∘
Анализ

Обратите внимание, что калькулятор будет возвращать только угол в квадрантах I или IV для функции синуса, поскольку это диапазон обратного синуса. Другой угол получается с помощью π − θ.π − θ.

Пример 9

Использование калькулятора для решения тригонометрического уравнения, содержащего секанс

Воспользуйтесь калькулятором, чтобы решить уравнение secθ = −4, secθ = −4, получив ответ в радианах.

Решение

Мы можем начать с некоторой алгебры.

secθ = −41cosθ = −4cosθ = −14secθ = −41cosθ = −4cosθ = −14

Убедитесь, что РЕЖИМ указан в радианах. Теперь используйте функцию обратного косинуса.

cos − 1 (−14) ≈1,8235 θ≈1,8235 + 2πkcos − 1 (−14) ≈1,8235 θ≈1,8235 + 2πk

Поскольку π2≈1,57π2≈1,57 и π≈3,14, π≈3,14, 1,8235 находится между этими двумя числами, поэтому θ≈1,8235θ≈1,8235 находится во втором квадранте. Косинус также отрицателен в квадранте III. Обратите внимание, что калькулятор возвращает только угол в квадрантах I или II для функции косинуса, поскольку это диапазон обратного косинуса.См. Рисунок 2.

Рисунок 2

Итак, нам также нужно найти меру угла в квадранте III. В квадранте III опорный угол равен θ’≈π − 1,8235≈1,3181. Θ’≈π − 1,8235≈1,3181. Другое решение в квадранте III: π + 1,3181≈4,4597.π + 1,3181≈4,4597.

Решения: 1.8235 ± 2πk1.8235 ± 2πk и 4.4597 ± 2πk.4.4597 ± 2πk.

Попробуй # 3

Решить cosθ = −0.2.cosθ = −0.2.

Решение тригонометрических уравнений в квадратичной форме

Решение квадратного уравнения может быть более сложным, но, опять же, мы можем использовать алгебру, как и любое квадратное уравнение.Посмотрите на схему уравнения. Есть ли в уравнении более одной тригонометрической функции или только одна? Какая тригонометрическая функция возводится в квадрат? Если представлена ​​только одна функция и один из членов возведен в квадрат, подумайте о стандартной форме квадратичной функции. Замените тригонометрическую функцию переменной, например xx или u.u. Если после подстановки уравнение выглядит как квадратное уравнение, то мы можем использовать те же методы решения квадратичных уравнений для решения тригонометрических уравнений.

Пример 10

Решение тригонометрического уравнения в квадратичной форме

Точно решите уравнение: cos2θ + 3cosθ − 1 = 0,0≤θ <2π.cos2θ + 3cosθ − 1 = 0,0≤θ <2π.

Решение

Начнем с подстановки и замены cos θθ на x.x. Нет необходимости использовать замену, но это может облегчить визуальное решение проблемы. Пусть cosθ = x.cosθ = x. У нас

Уравнение нельзя разложить на множители, поэтому мы будем использовать квадратную формулу x = −b ± b2−4ac2a.х = −b ± b2−4ac2a.

x = −3 ± (3) 2−4 (1) (- 1) 2 = −3 ± 132x = −3 ± (3) 2−4 (1) (- 1) 2 = −3 ± 132

Заменить xx с cosθ, cosθ и решить. Таким образом,

cosθ = −3 ± 132 θ = cos − 1 (−3 + 132) cosθ = −3 ± 132 θ = cos − 1 (−3 + 132)

Обратите внимание, что используется только знак +. Это связано с тем, что мы получаем ошибку, когда решаем θ = cos − 1 (−3−132) θ = cos − 1 (−3−132) на калькуляторе, поскольку область определения функции обратного косинуса равна [−1,1 ]. [- 1,1]. Однако есть второе решение:

cos − 1 (−3 + 132) ≈1,26 cos − 1 (−3 + 132) ≈1,26

Эта конечная сторона угла лежит в квадранте I.Поскольку косинус также положителен в квадранте IV, второе решение равно

2π − cos − 1 (−3 + 132) ≈5.022π − cos − 1 (−3 + 132) ≈5.02

Пример 11

Решение тригонометрического уравнения в квадратичной форме с помощью факторинга

Точно решите уравнение: 2sin2θ − 5sinθ + 3 = 0,0≤θ≤2π.2sin2θ − 5sinθ + 3 = 0,0≤θ≤2π.

Решение

Используя группировку, эту квадратичную величину можно разложить на множители. Либо сделайте настоящую замену, sinθ = u, sinθ = u, либо представьте ее, как мы множим:

2sin2θ − 5sinθ + 3 = 0 (2sinθ − 3) (sinθ − 1) = 0 2sin2θ − 5sinθ + 3 = 0 (2sinθ − 3) (sinθ − 1) = 0

Теперь установите каждый множитель равным нулю.

2sinθ − 3 = 0 2sinθ = 3 sinθ = 32 sinθ − 1 = 0 sinθ = 12sinθ − 3 = 0 2sinθ = 3 sinθ = 32 sinθ − 1 = 0 sinθ = 1

Затем найдите θ: sinθ ≠ 32, θ: sinθ ≠ 32, так как диапазон синусоидальной функции равен [−1,1]. [- 1,1]. Однако sinθ = 1, sinθ = 1, что дает решение π2.π2.

Анализ

Обязательно проверьте все решения в данном домене, так как некоторые факторы не имеют решения.

Попробуй # 4

Решите sin2θ = 2cosθ + 2,0≤θ≤2π.sin2θ = 2cosθ + 2,0≤θ≤2π.[Подсказка: сделайте замену, чтобы выразить уравнение только через косинус.]

Пример 12

Решение тригонометрического уравнения с помощью алгебры

Решите точно:

2sin2θ + sinθ = 0; 0≤θ <2π2sin2θ + sinθ = 0; 0≤θ <2π
Решение

Эта задача должна показаться вам знакомой, поскольку она похожа на квадратичную. Пусть sinθ = x.sinθ = x. Уравнение принимает вид 2×2 + x = 0,2×2 + x = 0. Начнем с факторинга:

2×2 + x = 0x (2x + 1) = 0 2×2 + x = 0x (2x + 1) = 0

Установите каждый коэффициент равным нулю.

x = 0 (2x + 1) = 0 x = −12 x = 0 (2x + 1) = 0 x = −12

Затем снова подставьте в уравнение исходное выражение sinθsinθ для x.x. Таким образом,

sinθ = 0 θ = 0, πsinθ = −12 θ = 7π6,11π6sinθ = 0 θ = 0, πsinθ = −12 θ = 7π6,11π6

Решения в области 0≤θ <2π0≤θ <2π равны 0, π , 7π6,11π6. 0, π, 7π6,11π6.

Если мы предпочитаем не заменять, мы можем решить уравнение, следуя той же схеме факторизации и установив каждый коэффициент равным нулю.

2sin2θ + sinθ = 0sinθ (2sinθ + 1) = 0 sinθ = 0 θ = 0, π 2sinθ + 1 = 0 2sinθ = −1 sinθ = −12 θ = 7π6,11π6 2sin2θ + sinθ = 0sinθ (2sinθ + 1) = 0 sinθ = 0 θ = 0, π 2sinθ + 1 = 0 2sinθ = −1 sinθ = −12 θ = 7π6,11π6
Анализ

Мы можем видеть решения на графике на рисунке 3. На интервале 0≤θ <2π, 0≤θ <2π график пересекает ось x- четыре раза в отмеченных решениях.Обратите внимание, что тригонометрические уравнения в квадратичной форме могут дать до четырех решений вместо ожидаемых двух, которые можно найти с помощью квадратных уравнений. В этом примере каждое решение (угол), соответствующее положительному значению синуса, даст два угла, которые приведут к этому значению.

Рисунок 3

Мы также можем проверить решения на единичном круге на Рисунке 2.

Пример 13

Решение тригонометрического уравнения, квадратичного по форме

Решите квадратное по форме уравнение: 2sin2θ − 3sinθ + 1 = 0,0≤θ <2π.2sin2θ − 3sinθ + 1 = 0,0≤θ <2π.

Решение

Мы можем факторизовать, используя группировку. Значения решения θθ можно найти на единичном круге:

(2sinθ − 1) (sinθ − 1) = 0 2sinθ − 1 = 0 sinθ = 12 θ = π6,5π6 sinθ = 1 θ = π2 (2sinθ − 1) (sinθ − 1) = 0 2sinθ − 1 = 0 sinθ = 12 θ = π6,5π6 sinθ = 1 θ = π2

Попробуй # 5

Решите квадратное уравнение 2cos2θ + cosθ = 0.2cos2θ + cosθ = 0.

Решение тригонометрических уравнений с использованием фундаментальных тождеств

Хотя алгебру можно использовать для решения ряда тригонометрических уравнений, мы также можем использовать фундаментальные тождества, потому что они упрощают решение уравнений. Помните, что методы, которые мы используем для решения проблем, не совпадают с методами проверки личности. Здесь применяются основные правила алгебры, а не переписывание одной стороны идентичности для соответствия другой стороне. В следующем примере мы используем два тождества, чтобы упростить уравнение.

Пример 14

Использование идентичностей для решения уравнения

Используйте тождества, чтобы точно решить тригонометрическое уравнение в интервале 0≤x <2π.0≤x <2π.

cosxcos (2x) + sinxsin (2x) = 32cosxcos (2x) + sinxsin (2x) = 32
Решение

Обратите внимание, что левая часть уравнения — это формула разности для косинуса.

cosxcos (2x) + sinxsin (2x) = 32 cos (x − 2x) = 32 Формула разности для косинуса cos (−x) = 32 Используйте тождество отрицательного угла.cosx = 32cosxcos (2x) + sinxsin (2x) = 32 cos (x − 2x) = 32 Формула разности для косинуса cos (−x) = 32 Используйте тождество отрицательного угла. cosx = 32

Из единичного круга на рисунке 2 мы видим, что cosx = 32cosx = 32, когда x = π6,11π6.x = π6,11π6.

Пример 15

Решение уравнения с использованием формулы двойного угла

Точно решите уравнение, используя формулу двойного угла: cos (2θ) = cosθ.cos (2θ) = cosθ.

Решение

У нас есть три варианта выражения для замены двойного угла косинуса. Поскольку проще решать одну тригонометрическую функцию за раз, мы выберем тождество с двойным углом, включающее только косинус:

cos (2θ) = cosθ 2cos2θ − 1 = cosθ 2cos2θ − cosθ − 1 = 0 (2cosθ + 1) (cosθ − 1) = 0 2cosθ + 1 = 0 cosθ = −12 cosθ − 1 = 0 cosθ = 1 cos (2θ ) = cosθ 2cos2θ − 1 = cosθ 2cos2θ − cosθ − 1 = 0 (2cosθ + 1) (cosθ − 1) = 0 2cosθ + 1 = 0 cosθ = −12 cosθ − 1 = 0 cosθ = 1

Итак, если cosθ = −12, cosθ = −12, тогда θ = 2π3 ± 2πkθ = 2π3 ± 2πk и θ = 4π3 ± 2πk; θ = 4π3 ± 2πk; если cosθ = 1, cosθ = 1, то θ = 0 ± 2πk.θ = 0 ± 2πk.

Пример 16

Решение уравнения с использованием идентификатора

Точно решите уравнение, используя тождество: 3cosθ + 3 = 2sin2θ, 0≤θ <2π.3cosθ + 3 = 2sin2θ, 0≤θ <2π.

Решение

Если мы перепишем правую часть, мы можем записать уравнение через косинус:

3 cosθ + 3 = 2 sin2θ3 cosθ + 3 = 2 (1 − cos2θ) 3 cosθ + 3 = 2−2cos2θ2cos2θ + 3 cosθ + 1 = 0 (2 cosθ + 1) (cosθ + 1) = 02 cosθ + 1 = 0cosθ = −12θ = 2π3,4π3cosθ + 1 = 0cosθ = −1θ = π3 cosθ + 3 = 2 sin2θ3 cosθ + 3 = 2 (1 − cos2θ) 3 cosθ + 3 = 2−2cos2θ2cos2θ + 3 cosθ + 1 = 0 (2 cosθ +1) (cosθ + 1) = 02 cosθ + 1 = 0cosθ = −12θ = 2π3,4π3cosθ + 1 = 0cosθ = −1θ = π

Наши решения: 2π3,4π3, π.2π3,4π3, π.

Решение тригонометрических уравнений с несколькими углами

Иногда невозможно решить тригонометрическое уравнение с тождествами, которые имеют кратный угол, например sin (2x) sin (2x) или cos (3x) .cos (3x). Столкнувшись с этими уравнениями, вспомните, что y = sin (2x) y = sin (2x) — это горизонтальное сжатие с коэффициентом 2 функции y = sinx.y = sinx. На интервале 2π, 2π мы можем изобразить два периода y = sin (2x), y = sin (2x), в отличие от одного цикла y = sinx.y = sinx.Такое сжатие графика приводит нас к мысли, что может быть вдвое больше x -перехватов или решений sin (2x) = 0sin (2x) = 0 по сравнению с sinx = 0. sinx = 0. Эта информация поможет нам решить уравнение.

Пример 17

Решение многоугольного тригонометрического уравнения

Решите точно: cos (2x) = 12cos (2x) = 12 на [0,2π). [0,2π).

Решение

Мы видим, что это уравнение является стандартным уравнением с углом, кратным углу.Если cos (α) = 12, cos (α) = 12, мы знаем, что αα находится в квадрантах I и IV. Хотя θ = cos − 112θ = cos − 112 даст решения только в квадрантах I и II, мы понимаем, что решения уравнения cosθ = 12cosθ = 12 будут в квадрантах I и IV.

Следовательно, возможные углы равны θ = π3θ = π3 и θ = 5π3.θ = 5π3. Итак, 2x = π32x = π3 или 2x = 5π3,2x = 5π3, что означает, что x = π6x = π6 или x = 5π6.x = 5π6. Имеет ли это смысл? Да, потому что cos (2 (π6)) = cos (π3) = 12. cos (2 (π6)) = cos (π3) = 12.

Есть ли другие возможные ответы? Вернемся к нашему первому шагу.

В квадранте I 2x = π3,2x = π3, поэтому x = π6x = π6, как указано. Давайте снова обратимся по кругу:

2x = π3 + 2π = π3 + 6π3 = 7π32x = π3 + 2π = π3 + 6π3 = 7π3

, поэтому x = 7π6.x = 7π6.

Еще один оборот дает

2x = π3 + 4π = π3 + 12π3 = 13π32x = π3 + 4π = π3 + 12π3 = 13π3

x = 13π6> 2π, x = 13π6> 2π, поэтому это значение для xx больше 2π, 2π, поэтому оно не решение на [0,2π). [0,2π).

В квадранте IV 2x = 5π3,2x = 5π3, поэтому x = 5π6x = 5π6, как указано. Давайте снова обратимся по кругу:

2x = 5π3 + 2π = 5π3 + 6π3 = 11π32x = 5π3 + 2π = 5π3 + 6π3 = 11π3

, поэтому x = 11π6.х = 11π6.

Еще один оборот дает

2x = 5π3 + 4π = 5π3 + 12π3 = 17π32x = 5π3 + 4π = 5π3 + 12π3 = 17π3

x = 17π6> 2π, x = 17π6> 2π, поэтому это значение для xx больше 2π, 2π, поэтому оно не решение на [0,2π). [0,2π).

Наши решения: π6,5π6,7π6, 11π6.π6,5π6,7π6 и 11π6. Обратите внимание, что всякий раз, когда мы решаем задачу в форме sin (nx) = c, sin (nx) = c, мы должны обойти единичный круг nn раз.

Решение задач прямоугольного треугольника

Теперь мы можем использовать все изученные нами методы для решения задач, связанных с применением свойств прямоугольных треугольников и теоремы Пифагора.Мы начнем с известной теоремы Пифагора, a2 + b2 = c2, a2 ​​+ b2 = c2, и смоделируем уравнение в соответствии с ситуацией.

Пример 18

Использование теоремы Пифагора для моделирования уравнения

Используйте теорему Пифагора и свойства прямоугольных треугольников, чтобы смоделировать уравнение, соответствующее задаче.

Один из тросов, которыми центр колеса обозрения «Лондонский глаз» крепится к земле, необходимо заменить. Центр колеса обозрения находится на высоте 69,5 метров над землей, а второй якорь на земле находится в 23 метрах от основания колеса обозрения.Примерно какой длины кабель и каков угол подъема (от земли до центра колеса обозрения)? См. Рисунок 4.

Рисунок 4

Решение

Используя предоставленную информацию, мы можем нарисовать прямоугольный треугольник. Мы можем найти длину кабеля с помощью теоремы Пифагора.

a2 + b2 = c2 (23) 2+ (69,5) 2≈5359 5359≈73,2 м a2 + b2 = c2 (23) 2+ (69,5) 2≈5359 5359≈73,2 м

Угол возвышения θ, θ, образованный вторым якорем на земле и тросом, идущим к центру колеса.Мы можем использовать касательную функцию, чтобы найти ее меру. Округлить до двух десятичных знаков.

tanθ = 69,523tan − 1 (69,523) ≈1,2522 ≈71,69∘ tanθ = 69,523tan − 1 (69,523) ≈1,2522 ≈71,69∘

Угол возвышения составляет примерно 71,7∘, 71,7∘, а длина кабеля составляет 73,2 метра. .

Пример 19

Использование теоремы Пифагора для моделирования абстрактной задачи

Правила безопасности OSHA требуют, чтобы основание лестницы располагалось на расстоянии 1 фута от стены на каждые 4 фута длины лестницы.Найдите угол, под которым лестница любой длины образует с землей, и высоту, на которой лестница касается стены.

Решение

Для лестницы любой длины основание должно находиться на расстоянии от стены, равном одной четвертой длины лестницы. Эквивалентно, если основание лестницы находится на расстоянии « а» футов от стены, длина лестницы будет 4 на футов. См. Рисунок 5.

Рис. 5

Сторона, примыкающая к θθ, равна a , а гипотенуза — 4a.4а. Таким образом,

cosθ = a4a = 14cos − 1 (14) ≈75,5∘ cosθ = a4a = 14cos − 1 (14) ≈75,5∘

Высота лестницы составляет 75,5∘75,5∘ с землей. Высота, на которой лестница касается стены, может быть найдена с помощью теоремы Пифагора:

a2 + b2 = (4a) 2 b2 = (4a) 2 − a2 b2 = 16a2 − a2 b2 = 15a2 b = 15aa2 + b2 = (4a) 2 b2 = (4a) 2 − a2 b2 = 16a2 − a2 b2 = 15a2 b = 15a

Таким образом, лестница касается стены на высоте 15a15a футов от земли.

7.Упражнения из 5 частей

Устные
1.

Всегда ли будут решения уравнений тригонометрических функций? Если нет, опишите уравнение, у которого не было бы решения. Объясните, почему да или почему нет.

2.

При решении тригонометрического уравнения, включающего более одной тригонометрической функции, всегда ли мы хотим попытаться переписать уравнение так, чтобы оно выражалось в терминах одной тригонометрической функции? Почему или почему нет?

3.

При решении линейных тригонометрических уравнений только с помощью синуса или косинуса, как мы узнаем, будут ли решения?

Алгебраический

Для следующих упражнений найдите все решения точно на интервале 0≤θ <2π.0≤θ <2π.

Для следующих упражнений решите точно на [0,2π). [0,2π).

19.

2cos (3θ) = — 22cos (3θ) = — 2

20.

cos (2θ) = — 32cos (2θ) = — 32

22.

2cos (π5θ) = 32cos (π5θ) = 3

Найдите все точные решения для следующих упражнений на [0,2π). [0,2π).

23.

сек (x) sin (x) −2sin (x) = 0sec (x) sin (x) −2sin (x) = 0

24.

tan (x) −2sin (x) tan (x) = 0tan (x) −2sin (x) tan (x) = 0

25.

2cos2t + cos (t) = 12cos2t + cos (t) = 1

26.

2tan2 (t) = 3сек (t) 2tan2 (t) = 3сек (t)

27.

2sin (x) cos (x) −sin (x) + 2cos (x) −1 = 02sin (x) cos (x) −sin (x) + 2cos (x) −1 = 0

30.

tan2 (x) = — 1 + 2tan (−x) tan2 (x) = — 1 + 2tan (−x)

31.

8sin2 (x) + 6sin (x) + 1 = 08sin2 (x) + 6sin (x) + 1 = 0

32.

tan5 (x) = tan (x) tan5 (x) = tan (x)

Для следующих упражнений решайте методами, показанными в этом разделе, точно на интервале [0,2π). [0,2π).

33.

sin (3x) cos (6x) −cos (3x) sin (6x) = — 0.9sin (3x) cos (6x) −cos (3x) sin (6x) = — 0.9

34.

sin (6x) cos (11x) −cos (6x) sin (11x) = — 0,1 sin (6x) cos (11x) −cos (6x) sin (11x) = — 0,1

35.

cos (2x) cosx + sin (2x) sinx = 1cos (2x) cosx + sin (2x) sinx = 1

36.

6sin (2t) + 9sint = 06sin (2t) + 9sint = 0

37.

9cos (2θ) = 9cos2θ − 49cos (2θ) = 9cos2θ − 4

40.

cos (6x) −cos (3x) = 0cos (6x) −cos (3x) = 0

Решите следующие упражнения точно на отрезке [0,2π). [0,2π). Если уравнения не учитываются, используйте формулу корней квадратного уравнения.

41.

tan2x − 3tanx = 0tan2x − 3tanx = 0

42.

sin2x + sinx − 2 = 0sin2x + sinx − 2 = 0

43.

sin2x − 2sinx − 4 = 0sin2x − 2sinx − 4 = 0

44.

5cos2x + 3cosx − 1 = 05cos2x + 3cosx − 1 = 0

45.

3cos2x − 2cosx − 2 = 03cos2x − 2cosx − 2 = 0

46. ​​

5sin2x + 2sinx − 1 = 05sin2x + 2sinx − 1 = 0

47.

tan2x + 5tanx − 1 = 0tan2x + 5tanx − 1 = 0

48.

cot2x = −cotxcot2x = −cotx

49.

−tan2x − tanx − 2 = 0 − tan2x − tanx − 2 = 0

Для следующих упражнений найдите точные решения на интервале [0,2π). [0,2π). Ищите возможности использовать тригонометрические тождества.

50.

sin2x − cos2x − sinx = 0sin2x − cos2x − sinx = 0

51.

sin2x + cos2x = 0sin2x + cos2x = 0

52.

sin (2x) −sinx = 0sin (2x) −sinx = 0

53.

cos (2x) −cosx = 0cos (2x) −cosx = 0

54.

2tanx2 − sec2x − sin2x = cos2x2tanx2 − sec2x − sin2x = cos2x

55.

1 − cos (2x) = 1 + cos (2x) 1 − cos (2x) = 1 + cos (2x)

57.

10sinxcosx = 6cosx10sinxcosx = 6cosx

58.

−3sint = 15costsint − 3sint = 15costsint

59.

4cos2x − 4 = 15cosx4cos2x − 4 = 15cosx

60.

8sin2x + 6sinx + 1 = 08sin2x + 6sinx + 1 = 0

61.

8cos2θ = 3−2cosθ8cos2θ = 3−2cosθ

62.

6cos2x + 7sinx − 8 = 06cos2x + 7sinx − 8 = 0

63.

12sin2t + cost − 6 = 012sin2t + cost − 6 = 0

Графический

Для следующих упражнений точно алгебраически определите все решения тригонометрического уравнения, затем проверьте результаты, построив уравнение на графике и найдя нули.

66.

6sin2x − 5sinx + 1 = 06sin2x − 5sinx + 1 = 0

67.

8cos2x − 2cosx − 1 = 08cos2x − 2cosx − 1 = 0

68.

100tan2x + 20tanx − 3 = 0100tan2x + 20tanx − 3 = 0

69.

2cos2x − cosx + 15 = 02cos2x − cosx + 15 = 0

70.

20sin2x − 27sinx + 7 = 020sin2x − 27sinx + 7 = 0

71.

2tan2x + 7tanx + 6 = 02tan2x + 7tanx + 6 = 0

72.

130tan2x + 69tanx − 130 = 0130tan2x + 69tanx − 130 = 0

Технологии

Для следующих упражнений используйте калькулятор, чтобы найти все решения до четырех знаков после запятой.

Для следующих упражнений решите уравнения алгебраически, а затем с помощью калькулятора найдите значения на интервале [0,2π).[0,2π). Округлить до четырех знаков после запятой.

77.

tan2x + 3tanx − 3 = 0tan2x + 3tanx − 3 = 0

78.

6tan2x + 13tanx = −66tan2x + 13tanx = −6

79.

tan2x − secx = 1tan2x − secx = 1

80.

sin2x − 2cos2x = 0sin2x − 2cos2x = 0

81.

2tan2x + 9tanx − 6 = 02tan2x + 9tanx − 6 = 0

82.

4sin2x + sin (2x) secx − 3 = 04sin2x + sin (2x) secx − 3 = 0

Расширения

Для следующих упражнений найдите все решения уравнений в точности на интервале [0,2π). [0,2π).

83.

csc2x − 3cscx − 4 = 0csc2x − 3cscx − 4 = 0

84.

sin2x − cos2x − 1 = 0sin2x − cos2x − 1 = 0

85.

sin2x (1 − sin2x) + cos2x (1 − sin2x) = 0sin2x (1 − sin2x) + cos2x (1 − sin2x) = 0

86.

3sec2x + 2 + sin2x − tan2x + cos2x = 03sec2x + 2 + sin2x − tan2x + cos2x = 0

87.

sin2x − 1 + 2cos (2x) −cos2x = 1sin2x − 1 + 2cos (2x) −cos2x = 1

88.

tan2x − 1 − sec3xcosx = 0tan2x − 1 − sec3xcosx = 0

89.

sin (2x) sec2x = 0sin (2x) sec2x = 0

90.

sin (2x) 2csc2x = 0sin (2x) 2csc2x = 0

91.

2cos2x − sin2x − cosx − 5 = 02cos2x − sin2x − cosx − 5 = 0

92.

1sec2x + 2 + sin2x + 4cos2x = 41sec2x + 2 + sin2x + 4cos2x = 4

Реальные приложения
93.

У самолета достаточно бензина, чтобы долететь до города в 200 милях к северо-востоку от его текущего местоположения. Если пилот знает, что город находится в 25 милях к северу, на сколько градусов к северу от востока должен лететь самолет?

94.

Если погрузочная рампа размещена рядом с грузовиком на высоте 4 фута, а ее длина составляет 15 футов, какой угол образует аппарель с землей?

95.

Если погрузочная рампа расположена рядом с грузовиком на высоте 2 фута, а ее длина составляет 20 футов, какой угол образует аппарель с землей?

96.

Женщина наблюдает за запуском ракеты, которая сейчас находится на высоте 11 миль. Если она стоит в 4 милях от стартовой площадки, под каким углом она смотрит вверх из горизонтали?

97.

Астронавт находится в запущенной ракете, которая сейчас находится на высоте 15 миль. Если мужчина стоит в 2 милях от стартовой площадки, под каким углом она смотрит на него сверху вниз из горизонтали? (Подсказка: это называется углом депрессии.)

98.

Женщина стоит в 8 метрах от 10-метрового здания.Под каким углом она смотрит на вершину здания?

99.

Мужчина стоит в 10 метрах от 6-метрового здания. Кто-то наверху здания смотрит на него сверху вниз. Под каким углом смотрит на него человек?

100.

У здания высотой 20 футов есть тень длиной 55 футов. Какой угол подъема солнца?

101.

У здания высотой 90 футов есть тень длиной 2 фута. Какой угол подъема солнца?

102.

Прожектор на земле в 3 метрах от человека ростом 2 метра отбрасывает 6-метровую тень на стену в 6 метрах от мужчины.Под каким углом свет?

103.

Прожектор на земле в 3 футах от женщины ростом 5 футов отбрасывает тень высотой 15 футов на стену в 6 футах от женщины. Под каким углом свет?

Для следующих упражнений найдите решение задачи со словом алгебраически. Затем воспользуйтесь калькулятором, чтобы проверить результат. Ответ округлите до десятых долей градуса.

104.

Человек выполняет стойку на руках, когда его ноги касаются стены, а руки находятся на расстоянии 1,5 фута от стены.Если рост человека 6 футов, какой угол у его ступни со стеной?

105.

Человек делает стойку на руках, при этом ноги касаются стены, а руки находятся на расстоянии 3 футов от стены. Если рост человека 5 футов, какой угол у его ступни со стеной?

106.

23-футовая лестница стоит рядом с домом. Если лестница соскользнет на расстоянии 7 футов от дома при недостаточном сцеплении с дорогой, какой угол должна составлять лестница относительно земли, чтобы избежать скольжения?

Домашняя страница | Yahoo Answers

1.Куриный суп с лапшой

Нет ничего неожиданного в том, что куриный суп с лапшой является лучшим из самых популярных супов в Америке. Мы в целом выросли с нашими людьми, которые нежно готовили большую тарелку куриного супа с лапшой, чтобы помочь нам почувствовать себя лучше в ужасный день. Теплый, ободряющий аромат возвращает нас к тем более простым случаям и помогает вспомнить дом.

2. Томатный суп

Тарелка томатного супа станет идеальным застольем. Независимо от того, подается ли он теплым с хрустящим жареным чеддером в холодное время года или холодным в виде пикантного гаспачо в конце весны, нет ничего неожиданного, этот новый детский суп станет последующим лучшим выбором Америки.Томатный суп невероятно адаптируем и обладает неповторимым вкусом, который все любят.

3. Похлебка из моллюсков

Похлебка из моллюсков — самая любимая территория Новой Англии, ее превосходный вкус стал настолько известен, что в настоящее время это американская норма. Независимо от того, предпочитаете ли вы образцовый крем на основе или более современные похлебки из моллюсков на томатной основе, завершенные скромным набором вафель из моллюсков, этот насыщенный суп — идеальная еда, чтобы согреться холодный, ненастный день независимо от того, где вы живете.

4. Картофельный суп

Картофельные супы к тому же удивительно гибкие, поэтому нет никаких сомнений в том, почему американцы их любят. Картофель и ветчина, картофель и лук-порей, тушеный картофель и гамбургеры — это лишь часть наших популярных супов номер один в Америке. Независимо от того, какого изысканного вкуса вы жаждете, есть картофельный суп, который станет идеальным дополнением к вашим предпочтениям.

5. Минестроне

Родом из Рима, Италия, американцы поспешили принять суп минестроне в качестве одного из своих лучших блюд.Вкусная смесь овощей и бобов в томатном бульоне невероятно сытна и имеет потрясающий вкус. Минестроне также является необычайно надежным решением, поскольку он невероятно адаптируется и может быть изготовлен независимо от сезона.

Нас не удивляет, что эти пять супов — лучший выбор Америки, поскольку мы любим их повсюду. Ваш суп №1 попал в список блокбастеров? Расскажите нам, мы будем рады получить от вас известие!

Решение тригонометрических уравнений | Precalculus II

Решение линейных тригонометрических уравнений с синусом и косинусом

Тригонометрические уравнения, как следует из названия, включают в себя тригонометрические функции.Во многом аналогично решению полиномиальных или рациональных уравнений, только определенные значения переменной будут решениями, если решения вообще есть. Часто мы решаем тригонометрическое уравнение на заданном интервале. Однако так же часто нас просят найти все возможные решения, и, поскольку тригонометрические функции являются периодическими, решения повторяются в течение каждого периода. Другими словами, тригонометрические уравнения могут иметь бесконечное количество решений. Кроме того, как и в случае с рациональными уравнениями, область определения функции должна быть рассмотрена, прежде чем мы предполагаем, что какое-либо решение является действительным.Период как синусоидальной функции, так и косинусной функции равен [latex] 2 \ pi [/ latex]. Другими словами, каждые [latex] 2 \ pi [/ latex] единицы, значения y- повторяются. Если нам нужно найти все возможные решения, мы должны добавить [latex] 2 \ pi k [/ latex], где [latex] k [/ latex] — целое число, к начальному решению. Вспомните правило, которое дает формат для указания всех возможных решений для функции, в которой период равен [latex] 2 \ pi: [/ latex]

[латекс] \ sin \ theta = \ sin \ left (\ theta \ pm 2k \ pi \ right) [/ latex]

Существуют аналогичные правила для указания всех возможных решений для других тригонометрических функций.Решение тригонометрических уравнений требует тех же методов, что и решение алгебраических уравнений. Мы читаем уравнение слева направо по горизонтали, как предложение. Мы ищем известные закономерности, множители, находим общие знаменатели и заменяем определенные выражения на переменные, чтобы упростить процесс решения. Однако с тригонометрическими уравнениями у нас также есть преимущество использования тождеств, которые мы разработали в предыдущих разделах.

Пример 1: Решение линейного тригонометрического уравнения с использованием функции косинуса

Найдите все возможные точные решения уравнения [latex] \ cos \ theta = \ frac {1} {2} [/ latex].

Решение

Из единичного круга мы знаем, что

[латекс] \ begin {array} {l} \ cos \ theta = \ frac {1} {2} \ hfill \\ \ text {} \ theta = \ frac {\ pi} {3}, \ frac {5 \ pi} {3} \ hfill \ end {array} [/ latex]

Это решения в интервале [латекс] \ left [0,2 \ pi \ right] [/ latex]. Все возможные решения дает

[латекс] \ theta = \ frac {\ pi} {3} \ pm 2k \ pi \ text {and} \ theta = \ frac {5 \ pi} {3} \ pm 2k \ pi [/ latex]

где [латекс] k [/ latex] — целое число.

Пример 2: Решение линейного уравнения с использованием синусоидальной функции

Найдите все возможные точные решения уравнения [latex] \ sin t = \ frac {1} {2} [/ latex].

Решение

Решение для всех возможных значений t означает, что решения включают углы за пределами периода [латекс] 2 \ pi [/ латекс]. Из единичного круга мы видим, что решениями являются [latex] t = \ frac {\ pi} {6} [/ latex] и [latex] t = \ frac {5 \ pi} {6} [/ latex] . Но проблема в том, чтобы указать все возможные значения, которые решают уравнение.Следовательно, ответ

[латекс] t = \ frac {\ pi} {6} \ pm 2 \ pi k \ text {и} t = \ frac {5 \ pi} {6} \ pm 2 \ pi k [/ latex]

где [латекс] k [/ latex] — целое число.

Как: решить тригонометрическое уравнение с помощью алгебры.

  • Найдите шаблон, который предлагает алгебраическое свойство, такое как разность квадратов или возможность разложения на множители.
  • Замените тригонометрическое выражение одной переменной, например [latex] x [/ latex] или [latex] u [/ latex].
  • Решите уравнение так же, как и алгебраическое уравнение.
  • Подставьте тригонометрическое выражение обратно вместо переменной в результирующих выражениях.
  • Найдите угол.

Пример 3: Решите тригонометрическое уравнение в линейной форме

Точно решите уравнение: [латекс] 2 \ cos \ theta -3 = -5,0 \ le \ theta <2 \ pi [/ latex].

Решение

Используйте алгебраические методы для решения уравнения.

[латекс] \ begin {array} {l} 2 \ cos \ theta -3 = -5 \ hfill \\ \ text {} 2 \ cos \ theta = -2 \ hfill \\ \ text {} \ cos \ theta = -1 \ hfill \\ \ text {} \ theta = \ pi \ hfill \ end {array} [/ latex]

Попробуй 1

Решите в точности следующее линейное уравнение на интервале [латекс] \ left [0,2 \ pi \ right): 2 \ sin x + 1 = 0 [/ latex].

Решение

Решение тригонометрических уравнений с помощью калькулятора

Не все функции могут быть решены точно с использованием только единичной окружности. Когда мы должны решить уравнение с углом, отличным от одного из специальных углов, нам понадобится калькулятор. Убедитесь, что он установлен на правильный режим, градусы или радианы, в зависимости от критериев данной проблемы.

Пример 8: Использование калькулятора для решения тригонометрического уравнения, содержащего синус

Воспользуйтесь калькулятором, чтобы решить уравнение [латекс] \ sin \ theta = 0.{\ circ} \ hfill \ end {array} [/ latex]

Анализ решения

Обратите внимание, что калькулятор будет возвращать только угол в квадрантах I или IV для функции синуса, поскольку это диапазон обратного синуса. Другой угол получается с помощью [латекса] \ pi — \ theta [/ latex].

Пример 9: Использование калькулятора для решения тригонометрического уравнения, содержащего секанс

Воспользуйтесь калькулятором, чтобы решить уравнение [латекс] \ сек \ тета = -4 [/ латекс], получив ответ в радианах.{-1} \ left (- \ frac {1} {4} \ right) \ приблизительно 1.8235 \ hfill \\ \ text {} \ theta \ приблизительно 1.8235 + 2 \ pi k \ hfill \ end {array} [/ latex ]

Поскольку [латекс] \ frac {\ pi} {2} \ приблизительно 1,57 [/ латекс] и [латекс] \ pi \ приблизительно 3,14 [/ латекс], 1,8235 находится между этими двумя числами, таким образом [латекс] \ theta \ приблизительно \ text {1} \ text {.8235} [/ latex] находится во втором квадранте. Косинус также отрицателен в квадранте III. Обратите внимание, что калькулятор возвращает только угол в квадрантах I или II для функции косинуса, поскольку это диапазон обратного косинуса.

Рисунок 2.

Итак, нам также нужно найти меру угла в квадранте III. В квадранте III опорный угол равен [латекс] \ theta \ text {} \ text {} \ text {‘} \ приблизительно \ pi — \ text {1} \ text {.8235} \ приблизительно \ text {1} \ text {.3181} \ text {.} [/ latex] Другое решение в квадранте III — [latex] \ theta \ text {} \ text {} \ text {‘} \ приблизительно \ pi + \ text {1} \ текст {.3181} \ приблизительно \ текст {4} \ text {.4597} \ text {.} [/ latex]

Растворы: [латекс] \ theta \ приблизительно 1.8235 \ pm 2 \ pi k [/ latex] и [латекс] \ theta \ приблизительно 4.4597 \ пм 2 \ пи к [/ латекс].

Попробуй 3

Решите [латекс] \ cos \ theta = -0.2 [/ latex].

Решение

Решение уравнений, содержащих одну тригонометрическую функцию

Когда нам задают уравнения, которые включают только одну из шести тригонометрических функций, их решения требуют использования алгебраических методов и единичного круга. Когда уравнение включает тригонометрические функции, отличные от синуса и косинуса, необходимо учитывать несколько факторов. Проблемы, связанные с величинами, обратными первичным тригонометрическим функциям, необходимо рассматривать с алгебраической точки зрения.Другими словами, мы напишем обратную функцию и найдем углы, используя эту функцию. Кроме того, уравнение, включающее функцию тангенса, немного отличается от уравнения, содержащего функцию синуса или косинуса. Во-первых, как мы знаем, период касательной равен [латекс] \ пи [/ латекс], а не [латекс] 2 \ пи [/ латекс]. Кроме того, область касательной — это все действительные числа, за исключением нечетных целых кратных [latex] \ frac {\ pi} {2} [/ latex], если, конечно, проблема не накладывает свои собственные ограничения на область.{2} \ theta} = \ pm \ sqrt {\ frac {1} {2}} \ hfill \\ \ text {} \ sin \ theta = \ pm \ frac {1} {\ sqrt {2}} = \ pm \ frac {\ sqrt {2}} {2} \ hfill \\ \ text {} \ theta = \ frac {\ pi} {4}, \ frac {3 \ pi} {4}, \ frac {5 \ pi} {4}, \ frac {7 \ pi} {4} \ hfill \ end {array} [/ latex]

Пример 5: Решение тригонометрического уравнения с использованием косеканса

Точно решите следующее уравнение: [latex] \ csc \ theta = -2,0 \ le \ theta <4 \ pi [/ latex].

Решение

Нам нужны все значения [latex] \ theta [/ latex], для которых [latex] \ csc \ theta = -2 [/ latex] в интервале [latex] 0 \ le \ theta <4 \ pi [/ latex] .

[латекс] \ begin {array} {l} \ csc \ theta = -2 \ hfill \\ \ frac {1} {\ sin \ theta} = — 2 \ hfill \\ \ sin \ theta = — \ frac { 1} {2} \ hfill \\ \ text {} \ theta = \ frac {7 \ pi} {6}, \ frac {11 \ pi} {6}, \ frac {19 \ pi} {6}, \ гидроразрыв {23 \ pi} {6} \ hfill \ end {array} [/ latex]

Анализ решения

Как [latex] \ sin \ theta = — \ frac {1} {2} [/ latex], обратите внимание, что все четыре решения находятся в третьем и четвертом квадрантах.

Пример 6: Решение уравнения с касательной

Точно решите уравнение: [латекс] \ tan \ left (\ theta — \ frac {\ pi} {2} \ right) = 1,0 \ le \ theta <2 \ pi [/ latex].

Решение

Напомним, что тангенциальная функция имеет период [латекс] \ пи [/ латекс]. На интервале [latex] \ left [0, \ pi \ right) [/ latex] и под углом [latex] \ frac {\ pi} {4} [/ latex] касательная имеет значение 1 Однако нам нужен угол [латекс] \ влево (\ theta — \ frac {\ pi} {2} \ right) [/ latex]. Таким образом, если [latex] \ tan \ left (\ frac {\ pi} {4} \ right) = 1 [/ latex], то

[латекс] \ begin {array} {c} \ theta — \ frac {\ pi} {2} = \ frac {\ pi} {4} \\ \ theta = \ frac {3 \ pi} {4} \ pm k \ pi \ end {array} [/ latex]

В интервале [латекс] \ left [0,2 \ pi \ right) [/ latex] у нас есть два решения:

[латекс] \ theta = \ frac {3 \ pi} {4} \ text {и} \ theta = \ frac {3 \ pi} {4} + \ pi = \ frac {7 \ pi} {4} [ / латекс]

Попробуй 2

Найдите все решения для [latex] \ tan x = \ sqrt {3} [/ latex].

Решение

Пример 7: Определите все решения уравнения, содержащего касательную

Определите все точные решения уравнения [латекс] 2 \ left (\ tan x + 3 \ right) = 5 + \ tan x, 0 \ le x <2 \ pi [/ latex].

Решение

Мы можем решить это уравнение, используя только алгебру. Выделите выражение [latex] \ tan x [/ latex] слева от знака равенства.

[латекс] \ begin {array} {cc} \ hfill 2 \ left (\ tan x \ right) +2 \ left (3 \ right) & = 5 + \ tan x \ hfill \\ \ hfill 2 \ tan x + 6 & = 5 + \ tan x \ hfill \\ \ hfill \ text {} 2 \ tan x- \ tan x & = 5-6 \ hfill \\ \ hfill \ tan x & = -1 \ hfill \ end {array} [ / латекс]

На единичной окружности есть два угла, значение касательной которых равно [latex] -1: \ theta = \ frac {3 \ pi} {4} [/ latex] и [latex] \ theta = \ frac {7 \ пи} {4} [/ латекс].

Решение тригонометрических уравнений в квадратичной форме

Решение квадратного уравнения может быть более сложным, но, опять же, мы можем использовать алгебру, как и любое квадратное уравнение. Посмотрите на схему уравнения. Есть ли в уравнении более одной тригонометрической функции или только одна? Какая тригонометрическая функция возводится в квадрат? Если представлена ​​только одна функция и один из членов возведен в квадрат, подумайте о стандартной форме квадратичной функции.{2} \ theta -5 \ sin \ theta + 3 = 0 \ hfill \\ \ left (2 \ sin \ theta -3 \ right) \ left (\ sin \ theta -1 \ right) = 0 \ hfill \ end {array} \\ [/ latex]

Теперь установите каждый коэффициент равным нулю.

[латекс] \ begin {array} {l} 2 \ sin \ theta -3 = 0 \ hfill \\ \ text {} 2 \ sin \ theta = 3 \ hfill \\ \ text {} \ sin \ theta = \ frac {3} {2} \ hfill \\ \ hfill \\ \ hfill \\ \ text {} \ sin \ theta -1 = 0 \ hfill \\ \ text {} \ sin \ theta = 1 \ hfill \ end { массив} \\ [/ latex]

Затем решите для [latex] \ theta: \ sin \ theta \ ne \ frac {3} {2} \\ [/ latex], поскольку диапазон синусоидальной функции равен [latex] \ left [-1,1 \ справа] \\ [/ латекс].{2} + x = 0 \ hfill \\ x \ left (2x + 1 \ right) = 0 \ hfill \ end {array} \\ [/ latex]

Установите каждый коэффициент равным нулю.

[латекс] \ begin {array} {l} \ text {} x = 0 \ text {} \ hfill \\ \ left (2x + 1 \ right) = 0 \ hfill \\ \ text {} x = — \ гидроразрыв {1} {2} \ hfill \ end {array} \\ [/ latex]

Затем снова подставьте в уравнение исходное выражение [latex] \ sin \ theta \\ [/ latex] вместо [latex] x \\ [/ latex]. Таким образом,

[латекс] \ begin {array} {l} \ sin \ theta = 0 \ hfill \\ \ text {} \ theta = 0, \ pi \ hfill \\ \ hfill \\ \ sin \ theta = — \ frac { 1} {2} \ hfill \\ \ text {} \ theta = \ frac {7 \ pi} {6}, \ frac {11 \ pi} {6} \ hfill \ end {array} \\ [/ latex]

Решения в области [latex] 0 \ le \ theta <2 \ pi \\ [/ latex]: [latex] \ theta = 0, \ pi, \ frac {7 \ pi} {6}, \ frac { 11 \ pi} {6} \\ [/ латекс].{2} \ theta + \ sin \ theta = 0 \ hfill \\ \ sin \ theta \ left (2 \ sin \ theta +1 \ right) = 0 \ hfill \\ \ text {} \ sin \ theta = 0 \ hfill \\ \ text {} \ theta = 0, \ pi \ hfill \\ \ hfill \\ \ text {} 2 \ sin \ theta + 1 = 0 \ hfill \\ \ text {} 2 \ sin \ theta = - 1 \ hfill \\ \ text {} \ sin \ theta = - \ frac {1} {2} \ hfill \\ \ text {} \ theta = \ frac {7 \ pi} {6}, \ frac {11 \ pi} {6} \ hfill \ end {array} \\ [/ latex]

Анализ решения

Мы можем видеть решения на графике на рисунке 3. На интервале [latex] 0 \ le \ theta <2 \ pi \\ [/ latex] график пересекает ось x- четыре раза в решениях отметил.Обратите внимание, что тригонометрические уравнения в квадратичной форме могут дать до четырех решений вместо ожидаемых двух, которые можно найти с помощью квадратных уравнений. В этом примере каждое решение (угол), соответствующее положительному значению синуса, даст два угла, которые приведут к этому значению.

Рисунок 3

Мы также можем проверить решения на единичном круге в тождествах суммы и разности.

Пример 13: Решение тригонометрического уравнения, квадратичного в форме

Решите квадратное по форме уравнение точно: [латекс] 2 {\ sin} ^ {2} \ theta -3 \ sin \ theta + 1 = 0,0 \ le \ theta <2 \ pi \\ [/ latex]. {2} \ theta + \ cos \ theta = 0 \\ [/ latex].

Решение

Решение тригонометрических уравнений с использованием основных тождеств

Хотя алгебру можно использовать для решения ряда тригонометрических уравнений, мы также можем использовать фундаментальные тождества, потому что они упрощают решение уравнений. Помните, что методы, которые мы используем для решения проблем, не совпадают с методами проверки личности. Здесь применяются основные правила алгебры, а не переписывание одной стороны идентичности для соответствия другой стороне. В следующем примере мы используем два тождества, чтобы упростить уравнение.

Пример 14: Использование идентичностей для решения уравнения

Используйте тождества, чтобы точно решить тригонометрическое уравнение в интервале [латекс] 0 \ le x <2 \ pi [/ latex].

[латекс] \ cos x \ cos \ left (2x \ right) + \ sin x \ sin \ left (2x \ right) = \ frac {\ sqrt {3}} {2} [/ latex]

Решение

Обратите внимание, что левая часть уравнения — это формула разности для косинуса.

[латекс] \ begin {array} {ll} \ hfill & \ hfill \\ \ cos x \ cos \ left (2x \ right) + \ sin x \ sin \ left (2x \ right) = \ frac {\ sqrt {3}} {2} \ hfill & \ hfill \\ \ text {} \ cos \ left (x — 2x \ right) = \ frac {\ sqrt {3}} {2} \ begin {array} {cccc} & & & \ end {array} \ hfill & \ text {Формула разности для косинуса} \ hfill \\ \ text {} \ cos \ left (-x \ right) = \ frac {\ sqrt {3}} {2} \ hfill & \ text {Использовать тождество отрицательного угла}.\ hfill \\ \ text {} \ cos x = \ frac {\ sqrt {3}} {2} \ hfill & \ hfill \ end {array} [/ latex]

Из единичного круга в суммах и различиях идентичностей мы видим, что [latex] \ cos x = \ frac {\ sqrt {3}} {2} [/ latex], когда [latex] x = \ frac {\ pi} { 6}, \ frac {11 \ pi} {6} [/ latex].

Пример 15: Решение уравнения с использованием формулы двойного угла

Точно решите уравнение, используя формулу двойного угла: [latex] \ cos \ left (2 \ theta \ right) = \ cos \ theta [/ latex].

Решение

У нас есть три варианта выражения для замены двойного угла косинуса.{2} \ theta — \ cos \ theta -1 = 0 \ hfill \\ \ left (2 \ cos \ theta +1 \ right) \ left (\ cos \ theta -1 \ right) = 0 \ hfill \\ \ текст {} 2 \ cos \ theta + 1 = 0 \ hfill \\ \ text {} \ cos \ theta = — \ frac {1} {2} \ hfill \\ \ hfill \\ \ text {} \ cos \ theta -1 = 0 \ hfill \\ \ text {} \ cos \ theta = 1 \ hfill \ end {array} [/ latex]

Итак, если [latex] \ cos \ theta = — \ frac {1} {2} [/ latex], то [latex] \ theta = \ frac {2 \ pi} {3} \ pm 2 \ pi k [ / latex] и [latex] \ theta = \ frac {4 \ pi} {3} \ pm 2 \ pi k [/ latex]; если [латекс] \ cos \ theta = 1 [/ latex], то [latex] \ theta = 0 \ pm 2 \ pi k [/ latex].{2} \ theta +3 cos \ theta + 1 & = 0 \ hfill \\ \ hfill \ left (2 cos \ theta +1 \ right) \ left (\ cos \ theta +1 \ right) & = 0 \ hfill \ \ \ hfill 2 cos \ theta + 1 & = 0 \ hfill \\ \ hfill \ cos \ theta & = — \ frac {1} {2} \ hfill \\ \ hfill \ theta & = \ frac {2 \ pi} { 3}, \ frac {4 \ pi} {3} \ hfill \\ \ hfill \ cos \ theta + 1 & = 0 \ hfill \\ \ hfill \ cos \ theta & = -1 \ hfill \\ \ hfill \ theta & = \ pi \ hfill \ end {array} [/ latex]

Наши решения: [latex] \ theta = \ frac {2 \ pi} {3}, \ frac {4 \ pi} {3}, \ pi [/ latex].

Решение тригонометрических уравнений с несколькими углами

Иногда невозможно решить тригонометрическое уравнение с тождествами, которые имеют несколько углов, например [латекс] \ sin \ left (2x \ right) [/ latex] или [latex] \ cos \ left (3x \ right) [/латекс].Столкнувшись с этими уравнениями, вспомните, что [latex] y = \ sin \ left (2x \ right) [/ latex] — это горизонтальное сжатие с коэффициентом 2 от функции [latex] y = \ sin x [/ латекс]. На интервале [latex] 2 \ pi [/ latex] мы можем построить график двух периодов [latex] y = \ sin \ left (2x \ right) [/ latex], в отличие от одного цикла [latex] y = \ sin x [/ латекс]. Такое сжатие графика приводит нас к мысли, что может быть вдвое больше x -перехватов или решений для [latex] \ sin \ left (2x \ right) = 0 [/ latex] по сравнению с [latex] \ sin x = 0 [/ латекс]. {- 1} \ frac {1} {2} [/ latex] дает решения только в квадрантах I и II, мы понимаем, что решения уравнения [latex] \ cos \ theta = \ frac {1} {2} [/ latex] будет в квадрантах I и IV.

Следовательно, возможные углы: [latex] \ theta = \ frac {\ pi} {3} [/ latex] и [latex] \ theta = \ frac {5 \ pi} {3} [/ latex]. Итак, [latex] 2x = \ frac {\ pi} {3} [/ latex] или [latex] 2x = \ frac {5 \ pi} {3} [/ latex], что означает, что [latex] x = \ frac {\ pi} {6} [/ latex] или [latex] x = \ frac {5 \ pi} {6} [/ latex]. Имеет ли это смысл? Да, потому что [латекс] \ cos \ left (2 \ left (\ frac {\ pi} {6} \ right) \ right) = \ cos \ left (\ frac {\ pi} {3} \ right) = \ гидроразрыв {1} {2} [/ latex].

Есть ли другие возможные ответы? Вернемся к нашему первому шагу.

В квадранте I [latex] 2x = \ frac {\ pi} {3} [/ latex], поэтому [latex] x = \ frac {\ pi} {6} [/ latex], как указано. Давайте снова обратимся по кругу:

[латекс] \ begin {array} {l} \ hfill \\ 2x = \ frac {\ pi} {3} +2 \ pi \ hfill \\ \ text {} = \ frac {\ pi} {3} + \ frac {6 \ pi} {3} \ hfill \\ \ text {} = \ frac {7 \ pi} {3} \ hfill \ end {array} [/ latex]

так [латекс] x = \ frac {7 \ pi} {6} [/ latex].

Еще один оборот дает

[латекс] \ begin {массив} {l} \ begin {array} {l} \\ 2x = \ frac {\ pi} {3} +4 \ pi \ end {array} \ hfill \\ \ text {} = \ frac {\ pi} {3} + \ frac {12 \ pi} {3} \ hfill \\ \ text {} = \ frac {13 \ pi} {3} \ hfill \ end {array} [/ latex ]

[latex] x = \ frac {13 \ pi} {6}> 2 \ pi [/ latex], поэтому это значение для [latex] x [/ latex] больше, чем [latex] 2 \ pi [/ latex] , так что это не решение для [латекса] \ left [0,2 \ pi \ right) [/ latex].

В квадранте IV [latex] 2x = \ frac {5 \ pi} {3} [/ latex], поэтому [latex] x = \ frac {5 \ pi} {6} [/ latex], как указано. Давайте снова обратимся по кругу:

[латекс] \ begin {array} {l} 2x = \ frac {5 \ pi} {3} +2 \ pi \ hfill \\ \ text {} = \ frac {5 \ pi} {3} + \ frac {6 \ pi} {3} \ hfill \\ \ text {} = \ frac {11 \ pi} {3} \ hfill \ end {array} [/ latex]

так [латекс] x = \ frac {11 \ pi} {6} [/ latex].

Еще один оборот дает

[латекс] \ begin {array} {l} 2x = \ frac {5 \ pi} {3} +4 \ pi \ hfill \\ \ text {} = \ frac {5 \ pi} {3} + \ frac {12 \ pi} {3} \ hfill \\ \ text {} = \ frac {17 \ pi} {3} \ hfill \ end {array} [/ latex]

[latex] x = \ frac {17 \ pi} {6}> 2 \ pi [/ latex], поэтому это значение для [latex] x [/ latex] больше, чем [latex] 2 \ pi [/ latex] , так что это не решение для [латекса] \ left [0,2 \ pi \ right) [/ latex].

Наши решения: [latex] x = \ frac {\ pi} {6}, \ frac {5 \ pi} {6}, \ frac {7 \ pi} {6}, \ text {и} \ frac {11 \ pi} {6} [/ латекс]. Обратите внимание, что всякий раз, когда мы решаем задачу в форме [латекс] \ sin \ left (nx \ right) = c [/ latex], мы должны обойти единичный круг [латекс] n [/ латекс] раз.

Решение задач прямоугольного треугольника

Теперь мы можем использовать все изученные нами методы для решения задач, связанных с применением свойств прямоугольных треугольников и теоремы Пифагора .{2} [/ latex] и смоделируйте уравнение в соответствии с ситуацией.

Пример 18: Использование теоремы Пифагора для моделирования уравнения

Используйте теорему Пифагора и свойства прямоугольных треугольников, чтобы смоделировать уравнение, соответствующее задаче.

Один из тросов, которыми центр колеса обозрения «Лондонский глаз» крепится к земле, необходимо заменить. Центр колеса обозрения находится на высоте 69,5 метров над землей, а второй якорь на земле находится в 23 метрах от основания колеса обозрения.{\ circ} [/ latex], а длина кабеля составляет 73,2 метра.

Пример 19: Использование теоремы Пифагора для моделирования абстрактной задачи

Правила безопасности OSHA требуют, чтобы основание лестницы располагалось на расстоянии 1 фута от стены на каждые 4 фута длины лестницы. Найдите угол, под которым лестница любой длины образует с землей, и высоту, на которой лестница касается стены.

Решение

Для лестницы любой длины основание должно находиться на расстоянии от стены, равном одной четвертой длины лестницы.{2} \ hfill \\ \ text {} b = \ sqrt {15} a \ hfill \ end {array} [/ latex]

Таким образом, лестница касается стены на [latex] \ sqrt {15} [/ latex] футах от земли.

  • При решении линейных тригонометрических уравнений мы можем использовать алгебраические методы так же, как при решении алгебраических уравнений. Ищите закономерности, такие как разница квадратов, квадратичная форма или выражение, которое хорошо поддается замене.
  • Уравнения, включающие одну тригонометрическую функцию, можно решить или проверить с помощью единичной окружности.
  • Мы также можем решать тригонометрические уравнения с помощью графического калькулятора.
  • Многие уравнения имеют квадратичную форму. Мы можем использовать подстановку, чтобы уравнение выглядело проще, а затем использовать те же методы, которые мы используем при решении алгебраической квадратичной системы: разложение на множители, квадратичная формула и т. Д.
  • Мы также можем использовать тождества для решения тригонометрического уравнения.
  • Мы можем использовать подстановку для решения многоугольного тригонометрического уравнения, которое является сжатием стандартной тригонометрической функции.Нам нужно будет учесть сжатие и убедиться, что мы нашли все решения на заданном интервале.
  • Реальные сценарии можно моделировать и решать с помощью теоремы Пифагора и тригонометрических функций.

Упражнения по разделам

1. Всегда ли будут решения уравнений тригонометрических функций? Если нет, опишите уравнение, у которого не было бы решения. Объясните, почему да или почему нет.

2. При решении тригонометрического уравнения, включающего более одной тригонометрической функции, всегда ли мы хотим попытаться переписать уравнение так, чтобы оно выражалось в терминах одной тригонометрической функции? Почему или почему нет?

3.Решая линейные тригонометрические уравнения только с помощью синуса или косинуса, как мы узнаем, будут ли решения?

Для следующих упражнений найдите все решения точно на интервале [latex] 0 \ le \ theta <2 \ pi [/ latex].

4. [латекс] 2 \ sin \ theta = — \ sqrt {2} [/ латекс]

5. [латекс] 2 \ sin \ theta = \ sqrt {3} [/ латекс]

6. [латекс] 2 \ cos \ theta = 1 [/ латекс]

7. [латекс] 2 \ cos \ theta = — \ sqrt {2} [/ латекс]

8. [латекс] \ tan \ theta = -1 [/ латекс]

9.{2} x — 4 = 0 [/ латекс]

Для следующих упражнений решите точно на [латексе] \ левом [0,2 \ пи \ правом) [/ латексе].

13. [латекс] 2 \ cos \ theta = \ sqrt {2} [/ латекс]

14. [латекс] 2 \ cos \ theta = -1 [/ латекс]

15. [латекс] 2 \ sin \ theta = -1 [/ латекс]

16. [латекс] 2 \ sin \ theta = — \ sqrt {3} [/ латекс]

17. [латекс] 2 \ sin \ left (3 \ theta \ right) = 1 [/ латекс]

18. [латекс] 2 \ sin \ left (2 \ theta \ right) = \ sqrt {3} [/ latex]

19. [латекс] 2 \ cos \ left (3 \ theta \ right) = — \ sqrt {2} [/ latex]

20.[латекс] \ cos \ left (2 \ theta \ right) = — \ frac {\ sqrt {3}} {2} [/ latex]

21. [латекс] 2 \ sin \ left (\ pi \ theta \ right) = 1 [/ латекс]

22. [латекс] 2 \ cos \ left (\ frac {\ pi} {5} \ theta \ right) = \ sqrt {3} [/ latex]

Для следующих упражнений найдите все точные решения на [latex] \ left [0,2 \ pi \ right) [/ latex].

23. [латекс] \ sec \ left (x \ right) \ sin \ left (x \ right) -2 \ sin \ left (x \ right) = 0 [/ латекс]

24. [латекс] \ загар \ влево (х \ вправо) -2 \ грех \ влево (х \ вправо) \ загар \ влево (х \ вправо) = 0 [/ латекс]

25.{5} \ left (x \ right) = \ tan \ left (x \ right) [/ латекс]

Для следующих упражнений решайте методами, показанными в этом разделе, точно в интервале [латекс] \ left [0,2 \ pi \ right) [/ latex].

33. [латекс] \ sin \ left (3x \ right) \ cos \ left (6x \ right) — \ cos \ left (3x \ right) \ sin \ left (6x \ right) = — 0,9 [/ латекс]

34. [латекс] \ sin \ left (6x \ right) \ cos \ left (11x \ right) — \ cos \ left (6x \ right) \ sin \ left (11x \ right) = — 0,1 [/ латекс]

35. [латекс] \ cos \ left (2x \ right) \ cos x + \ sin \ left (2x \ right) \ sin x = 1 [/ latex]

36.{2} x + 69 \ tan x — 130 = 0 [/ латекс]

Для следующих упражнений используйте калькулятор, чтобы найти все решения до четырех знаков после запятой.

73. [латекс] \ sin x = 0,27 [/ латекс]

74. [латекс] \ sin x = -0,55 [/ латекс]

75. [латекс] \ tan x = -0,34 [/ латекс]

76. [латекс] \ cos x = 0,71 [/ латекс]

Для следующих упражнений решите уравнения алгебраически, а затем с помощью калькулятора найдите значения в интервале [latex] \ left [0,2 \ pi \ right) [/ latex]. Округлить до четырех знаков после запятой.{2} x = 4 [/ латекс]

93. У самолета достаточно бензина, чтобы долететь до города в 200 милях к северо-востоку от его текущего местоположения. Если пилот знает, что город находится в 25 милях к северу, на сколько градусов к северу от востока должен лететь самолет?

94. Если погрузочная рампа расположена рядом с грузовиком на высоте 4 фута, а ее длина составляет 15 футов, то какой угол образует аппарель с землей?

95. Если погрузочная рампа расположена рядом с грузовиком на высоте 2 фута, а ее длина составляет 20 футов, то какой угол образует аппарель с землей?

96.Женщина наблюдает за запущенной ракетой, которая сейчас находится на высоте 11 миль. Если она стоит в 4 милях от стартовой площадки, под каким углом она смотрит вверх из горизонтали?

97. Космонавт находится в запущенной ракете на высоте 15 миль. Если мужчина стоит в 2 милях от стартовой площадки, под каким углом она смотрит на него сверху вниз из горизонтали? (Подсказка: это называется углом депрессии.)

98. Женщина стоит в 8 метрах от 10-метрового здания. Под каким углом она смотрит на вершину здания?

99.Мужчина стоит в 10 метрах от 6-метрового дома. Кто-то наверху здания смотрит на него сверху вниз. Под каким углом смотрит на него человек?

100. У здания высотой 20 футов есть тень длиной 55 футов. Какой угол подъема солнца?

101. У здания высотой 90 футов есть тень длиной 2 фута. Какой угол подъема солнца?

102. Прожектор на земле в 3 метрах от человека ростом 2 метра отбрасывает 6-метровую тень на стену в 6 метрах от человека.Под каким углом свет?

103. Прожектор на земле в 3 футах от женщины 5 футов высотой отбрасывает тень 15 футов высотой на стену в 6 футах от женщины. Под каким углом свет?

Для следующих упражнений найдите решение следующей задачи со словами алгебраически. Затем воспользуйтесь калькулятором, чтобы проверить результат. Ответ округлите до десятых долей градуса.

104. Человек выполняет стойку на руках, касаясь ступней стены и рук на расстоянии 1,5 фута от стены.Если рост человека 6 футов, какой угол у его ступни со стеной?

105. Человек выполняет стойку на руках, при этом ноги касаются стены, а руки находятся на расстоянии 3 футов от стены. Если рост человека 5 футов, какой угол у его ступни со стеной?

106. Рядом с домом стоит 23-футовая лестница. Если лестница соскользнет на расстоянии 7 футов от дома при недостаточном сцеплении с дорогой, какой угол должна составлять лестница относительно земли, чтобы избежать скольжения?

Тригонометрические уравнения — Учебные материалы для IIT JEE

Уравнение, включающее одно или несколько тригонометрических отношений неизвестного угла, называется тригонометрическим уравнением e.грамм. cos 2 x — 4 sin x = 1

Следует отметить, что тригонометрическое тождество выполняется для каждого значения неизвестного угла, тогда как тригонометрическое уравнение выполняется только для некоторых значений (конечных или бесконечных) неизвестного угла.

например sec 2 x — tan 2 x = 1 является тригонометрическим тождеством, поскольку оно выполняется для любого значения x Î R.

РЕШЕНИЕ ТРИГОНОМЕТРИЧЕСКОГО УРАВНЕНИЯ

Значение неизвестного угла, которое удовлетворяет данному уравнению, называется решением уравнения e.грамм. грех q = ½ Þq = p / 6.

Общее решение

Поскольку тригонометрические функции являются периодическими функциями, решения тригонометрических уравнений могут быть обобщены с помощью периодичности тригонометрических функций. Решение, состоящее из всевозможных решений тригонометрического уравнения, называется его общим решением.

Мы используем следующие формулы для решения тригонометрических уравнений:

· грех q = 0 Þ q = np,

· cos q = 0 Þq = (2n + 1),

· tan q = 0 Þ q = np,

· sin q = sin a Þq = np + (–1) n a, где aÎ [–p / 2, p / 2]

· cos q = cos aÞq = 2np ± a, где aÎ [0, p]

· tan q = tan a Þ q = np + a, где aÎ (–p / 2, p / 2)

· sin 2 q = sin 2 a, cos 2 q = cos 2 a, tan 2 q = tan 2 aÞq = np ± a,

· грех q = 1 Þq = (4n + 1),

· cos q = 1 Þ q = 2np,

· cos q = –1 Þ q = (2n + 1) p,

· sin q = sin a и cos q = cos aÞ q = 2np + a.

Примечание:

· Повсюду в этой главе n принимается как целое число, если не указано иное.

· Необходимо дать общее решение, если только решение не требуется в заданном интервале.

За главное значение угла принимается

· a. Численно наименьший угол называется главным значением.

Метод определения главного значения

Предположим, мы должны найти главное значение, удовлетворяющее уравнению sin = -.

Так как sin отрицателен, будет в 3-м или 4-м квадранте. Мы можем подойти к 3-му или 4-му квадранту с двух сторон. Если мы возьмем направление против часовой стрелки, числовое значение угла будет больше чем. Если подойти к нему по часовой стрелке, угол будет численно меньше, чем. За главное значение мы должны взять численно наименьший угол.

Итак, для основного значения:

1. Если угол находится в 1-м или 2-м квадранте, мы должны выбрать направление против часовой стрелки, а если угол, если угол находится в 3-м или 4-м квадранте, мы должны выбрать направление по часовой стрелке.

2. Главное значение никогда не может быть численно больше, чем.

3. Главное значение всегда находится в первом круге (т.е. в первом повороте)

По вышеуказанным критериям будет или. Среди этих двух наименьшее числовое значение. Отсюда главное значение удовлетворения уравнения sin = -.

Алгоритм нахождения главного аргумента:

Шаг 1: Сначала нарисуйте тригонометрический круг и отметьте квадрант, в котором может находиться угол.

Шаг 2: Выберите направление против часовой стрелки для 1-го и 2-го квадрантов и выберите направление по часовой стрелке для 3-го и 4-го квадрантов.

Шаг 3: Найдите угол при первом повороте.

Шаг 4: Выберите численно наименьший угол среди этих двух значений. Найденный таким образом угол будет главной величиной.

Шаг 5: В случае, если два угла, один с положительным знаком, а другой с отрицательным знаком, соответствуют численно наименьшему углу, тогда принято выбирать угол с положительным знаком в качестве главного значения.

Пример 1: Iftan = — 1, то будет находиться во 2-м или 4-м квадранте.

Для 2-го квадранта мы выберем против часовой стрелки, а для 4-го квадранта. мы выберем направление по часовой стрелке.

В первом кружке получены два значения и.

Среди этих двух углов является наименьшим в численном отношении. Следовательно, главное значение.

Пример 2: Если cos =, то будет лежать в 1 или 4 -м квадранте .

Для 1-го квадранта мы выберем направление против часовой стрелки, а для 4-го квадранта мы выберем направление по часовой стрелке.

Таким образом, в первом круге находятся два значения и.

Оба и — имеют одинаковое числовое значение. В таком случае будет выбрано главное значение.

Рисунок 17: Решить кроватку (sinx + 3) = 1.

Решение: sinx + 3 = Þ Þ n = 1 Þ sinx =

Þ x = или

Иллюстрация 18: Если sin 5x + sin 3x + sin x = 0, то найдите значение x, отличное от нуля, лежащее между 0 £ x £ .

Решение: sin 5x + sin 3x + sin x = 0 Þ (sin 5x + sin x) + sin 3x = 0

Þ 2 sin 3x cos 2x + sin 3x = 0 Þ sin 3x (2 cos 2x + 1) = 0

Þ sin 3x = 0; cos 2x = — Þ 3x = np, 2x = 2np ±

Требуемое значение x равно.

Рисунок 19: Найти весь острый угол a такой, что cos a cos 2 a cos = .

Решение: Дано, что cosa cos2a cos4a =

Þ 2sina cosa cos2a cos4a = Þ 2sin2a cos2a cos4a =

Þ 2sin4a cos4a = sinaÞ sin8a — sina = 0

Þ 2sincos = 0

Либо sin = 0 Þ Þa =

Для n = 0 a = 0, что не является решением.

Þa = n = 1, т.е. a =

или cos Þ = (2n + 1) Þa = (2n + 1) Þa =

Следовательно, a =.

Рисунок 20. Решите для x: .

Решение:

Þ

Þ

Þ ÞÞ

sin2x = ± 1 Þ 2x = (2n + 1) Þ x = (2n + 1), n ​​I

ОБЪЕКТИВНОЕ ЗАДАНИЕ

1: Общее значение q, удовлетворяющее обоим, составляет:

(А) 2нп (Б) 2нп + 7п / 6

(В) нп + п / 4 (Д) 2нп + п / 4

Решение: Давайте сначала выясним, что q лежит между 0 и 360 °.

Так как Þq = 210 ° или 330 °

и Þq = 30 ° или 210 °

Следовательно, q = 210 ° или значение, удовлетворяющее обоим требованиям.

\ Общее значение

Следовательно, (B) — правильный ответ.

2: Ö3 сек20 ° — сек20 ° =

(А) 1 (В) 2

(К) 3 (Г) 4

Решение: Дано =

=

Следовательно (D) — правильный ответ.

3: загар A + 2 загар 2A + 4 загар 4A + 8 кроватка 8A =

(A) Детская кроватка A (B) желто-коричневый 6A

(C) детская кроватка 4A (D) Ни один из этих

Раствор: загар A + 2 загар 2A + 4 загар 4A + 8 кроватка 8A

= tanA + 2tan2A + 4tan4A + 8

= детская кроватка

Следовательно, (A) — правильный ответ.

4: Значение греха 12 °. sin48 ° .sin54 ° =

(А) 1/8 (В) 1/6

(C) 1/4 (D) 1/2

Решение: sin 12 °. sin48 ° .sin54 ° =

=

=

=

=

Альтернативный метод

Пусть q = 12 °

sin 12 °.sin48 ° .sin54 ° =

=

Следовательно, (A) — правильный ответ.

5: Наименьшее положительное значение x (в градусах), для которого

загар (x + 100 °) = загар (x + 50 °) загар x загар (x — 50 °) составляет:

(А) 30 ° (В) 45 °

(С) 60 ° (Г) 90 °

Решение: Отношение можно записать как

Þ

Þ

Þ Þ cos50 ° + 2sin (2x + 50 °) cos (2x + 50 °) = 0

cos50 ° + sin (4x + 100 °) = 0 Þ cos50 ° + cos (4x + 10 °) = 0

cos (2x + 30 °) cos (2x — 20 °) = 0 Þ x = 30 °, 55 °

Þ Наименьшее значение x = 30 °

Следовательно, (A) — правильный ответ.

6. Наиболее общее значение q, удовлетворяющее 3 — 2cosq –4sinq –cos2q + sin2q = 0:

(А) 2нп (Б) 2нп + п / 2

(К) 4нп (Д) 2нп + п / 4

Решение: 3 — 2cos q — 4 sin q — cos 2q + sin 2q = 0

Þ 3 — 2cos q — 4 sin q — 1 + 2sin 2 q + 2sin q cos q = 0

Þ 2sin 2 q — 2cosq — 4sin q + 2sin q cos q + 2 = 0

Þ (sin 2 q — 2sin q + 1) + cos q (sinq — 1) = 0

Þ (sin q — 1) [sin q — 1 + cos q] = 0

либо sin q = 1

Þq = 2np + p / 2, где n Î I

или, sin q + cos q = 1

cos (q — p / 4) = cos (p / 4) Þq — p / 4 = 2np ± p / 4

Þ q = 2np, 2np + p / 2, где n Î I

Следовательно, q = 2np, 2np + p / 2.

Следовательно (A, B) — правильный ответ.

7: Если sinq = 3sin (q + 2a), то значение tan (q + a) + 2tana равно:

(А) 0 (В) 2

(К) 4 (Г) 1

Решение: дан sin q = 3sin (q + 2a)

Þ грех (д + а-а) = 3 син (д + а + а)

Þ sin (q + a) cosa — cos (q + a) sina

= 3sin (q + a) cosa + 3cos (q + a) sina

Þ –2sin (q + a) cosa = 4cos (q + a) sina

Þ

Þ загар (д + а) + 2тана = 0

Следовательно, (A) — правильный ответ.

8: Минимальное значение 3tan 2 q + 12 cot 2 q:

(А) 6 (В) 8

(C) 10 (D) Ни один из этих

Решение: A.M. ³ G.M Þ (3tan 2 q +12 детская кроватка 2 q) ³ 6

Þ 3 tan 2 q + 12cot 2 q имеет минимальное значение 12.

Следовательно (D) — правильный ответ.

9: Если A + B + C =, то значение tanA + tanB + tanC равно:

(А) 3 (В) 2

(C)> 3 (D)> 2

Решение: tan (A + B) = tan (- C)

или, = tanC

или, tanA + tanB + tanC = tana tanB tanC

[с А.M. G.M.]

или, tanA tanB tanC

или, A B C 27 [в кубе с обеих сторон]

или tanA tanB tanC 3

tanA + tanB + tanC 3.

Следовательно, (A) — правильный ответ.

10: Пусть 0

(А) (В)

(C) (D) Ничего из этого.

Решение: Из второго уравнения имеем

sin2B = sin2A… (1)

и от первого равенства

3A = 1-2 B = cos2B… (2)

Теперь cos (A + 2B) = cosA. cos2B — sinA. sin2B

= 3 cosA. А -. sinA. sin2A

= 3cosA. А — 3А.cosA = 0

A + 2B = или

При условии, что 0

Следовательно, A + 2B =.

Следовательно, (C) — правильный ответ.

11: Если a cos 3 q + 3a cos q sin 2 q = x и a sin 3 q + 3a cos 2 q sin q = y, то (x + y) 2/3 + (х — у) 2/3 =

(A) 2a 2/3 (B) a 2/3

(C) 3a 2/3 (D) 2a 1/3

Решение: a cos 3 q + 3a cos q sin 2 q = x

a sin 3 q + 3a cos 2 q sin q = y

x + y = a [sin 3 q + cos 3 q + 3 sin q cos q (sin q + cos q)] = a (sinq + cosq) 3

= грех q + cos q …… (1)

x — y = a [cos 3 q — sin 3 q + 3 cosq sin 2 q — 3 cos 2 q sin q] = a [cosq — sinq] 3

= cos q — sin q …… (2)

(sin q + cos q) 2 + (cos q — sin q) 2 =

2 (sin 2 q + cos 2 q) =

(x + y) 2/3 + (x — y) 2/3 = 2a 2/3 .

Следовательно, (A) — правильный ответ.

12: Если, то sin4a =

(А) а / 2 (В) а

(C) a 2/3 (D) 2a

Решение: Пусть a = sin 4qÞ = cos 2q + sin 2q и = cos 2q — sin 2q

(1 +) загар а = (1 +)

Þ (1 + cos 2q + sin 2q) tan a = 1 + cos 2q — sin 2q

Þ = детская кроватка

Þ = детская кроватка Þ

Þ загар = загар Þq =

Þ a = sin 4q = sin (p — 4a) = sin 4 a

Следовательно, (B) — правильный ответ.

13: Если cos 2 q = и tan 2 = tan 2/3 a, то cos 2/3 a + sin 2/3 a =

(А) 2а 2/3 (В)

(C) (D) 2a 1/3

Решение: cos 2 q =, tan 2 = tan 2/3 a

загар 3 = загар aÞ

= к

sin 3 = k sin a …… (1)

cos 3 = k cos a …… (2)

k 2/3 sin 2/3 a + k 2/3 cos a = 1

sin 2/3 a + cos 2/3 a =

Возведение в квадрат и сложение (1) и (2)

k 2 (sin 2 a + cos 2 a) = sin 6 + cos 6 =

к 2 = 1 — sin 2 q = 1 — + cos 2 q

k 2 = Þ k =

sin 2/3 a + cos 2/3 a =.

Следовательно, (B) — правильный ответ.

14: Если 3 sin 2 a + 2 sin 2 b = 1 и 3 sin 2a –2 sin 2b = 0, где a, b — положительные острые углы, то a + 2b =

(А) (В)

(К) (Г)

Решение: 3 sin 2 a + 2 sin 2 b = 1 …… (1)

3 греха 2a = 2 греха 2b …… (2)

3 sin 2 a = 1-2 sin 2 b = cos 2b

3 sin a sin a = cos 2b …… (3)

из уравнения (2)

3.2 sin a cos a = 2 sin 2b

3 греха =

из уравнения (3)

sin a = cos 2b

cos a cos 2b — sin a sin 2b = 0

cos (a + 2b) = 0

Þa + 2b =.

Следовательно, (A) — правильный ответ.

15: Значение:

(А) (В)

(К) (Г)

Решение:

=

=

=

Следовательно, (A) — правильный ответ.

16: количество решений sin 3 x cos x + sin 2 x cos 2 x + sin x cos 3 x = 1 в [0, 2p] равно

(А) 4 (В) 2

(К) 1 (Д) 0

Решение: sin x cos x [sin 2 x + sin x cos x + cos 2 x] = 1

Þ sin x cos x + (sin x cos x) 2 = 1

sin 2 2x + 2 sin 2x –4 = 0 Þ sin 2x =, что невозможно.

Следовательно (D) — правильный ответ.

17: Количество решений уравнения x 3 + 2x 2 + 5x + 2cosx = 0 в
[0, 2p]:

(А) 0 (В) 1

(К) 2 (Г) 3

Решение: Пусть f (x) = x 3 + 2x 2 + 5x +2 cosx

Þ f ¢ (x) = 3x 2 + 4x + 5-2 sinx

= 3

Now «x (as -1 £ sinx £ 1)

Þ f ¢ (x)> 0 «x

Þ f (x) — возрастающая функция.

Теперь f (0) = 2

Þ f (x) = 0 не имеет решения в [0, 2p].

Следовательно, (A) — правильный ответ.

18: значение равно

.

(А) -1 (В)

(К) (Г)

Решение:.

Следовательно (D) — правильный ответ.

19: sinnx =, где n — нечетное натуральное число, тогда:

(A) = 1, = 2n (B) = 1, = n

(C) = 0, = n (D) = 0, = -n

Решение: sin nx = Im (e in x ) = Im ((cosx + i sinx) n )

…..

Поскольку n нечетное, пусть n = 2 + 1

sin nx = — +….

= — + +….

=

Следовательно, (C) — правильный ответ.

20: Если tanx = n. tany, n, то максимальное значение (x — y) равно:

(А) (В)

(К) (Г)

Решение: tanx = n tany, cos (x — y)

= cosx.уютный + sinx.siny.

cos (x — y) = cosx.cosy (1 + tanx.tany)

= cosx. уютный (1 + п загар 2 л)

Сейчас,

Следовательно (D) — правильный ответ.

21: Если 3sinq + 5cosq = 5, то значение 5sinq — 3cosq равно

(А) 5 (В) 3

(C) 4 (D) ни один из этих

Решение: 3sinq = 5 (1 — cosq) = 5 ´ 2sin 2 q / 2 Þ tanq / 2 = 3/5

5sinq — 3cosq = =

Следовательно, (B) — правильный ответ.

22: В DABC, если cotA cotB cotC> 0, то D равно

(A) остроугольный (B) прямоугольный

(C) тупоугольный (D) не существует

Решение: Поскольку cotA cotB cotC> 0

cotA, cotB, cotC положительные ÞD остроугольный

Следовательно, (A) — правильный ответ.

23: Если p <2q <, то равно

(A) –2cosq (B) –2sinq

(C) 2cosq (D) 2sinq

Решение: =

= 2 | sinq | = 2sinq как

Следовательно, (D) — правильный ответ.

24: Если tanq = для некоторого неквадратного натурального числа n, то sec2q равно

(A) рациональное число (B) иррациональное число

(C) положительное число (D) ни один из этих

Решение:

где n — неквадратное натуральное число, поэтому 1 — n ¹ 0.

Þ sec2q — рациональное число.

Следовательно, (A) — правильный ответ.

25: минимальное значение cos (cosx) составляет

(A) 0 (B) –cos1

(C) cos1 (D) –1

Решение: cos x изменяется от –1 до 1 для всех действительных x.

Таким образом, cos (cosx) изменяется от cos1 до cos0 Þ минимальное значение cos (cosx) равно cos1.

Следовательно, (C) — правильный ответ.

26: Если sin x cos y = 1/4 и 3 tan x = 4 tan y, то найдите значение sin (x + y).

(А) 1/16 (В) 7/16

(C) 5/16 (D) ни один из этих

Решение: 3 tan x = 4 tan y Þ 3 sin x cos y = 4 cos x sin y

Þ 3/4 = 4 cos x sin y Þ cos x sin y = 3/16

\ грех (х + у) знак равно грех х соз у + соз х грех у =.

Следовательно, (B) — правильный ответ.

27: максимальное значение 4sin 2 x + 3cos 2 x + равно

(А) (В)

(К) 9 (Г) 4

Решение: максимальное значение 4sin 2 x + 3cos 2 x, т.е. sin 2 x + 3 равно 4, а sin + cos равно =, оба значения достигаются при x = p / 2.Следовательно, данная функция имеет максимальное значение

Следовательно, (A) — правильный ответ.

28: Если a и b являются решениями sin 2 x + a sin x + b = 0, а также решения cos 2 x + c cos x + d = 0, то sin (a + b) равен равно

(А) (В)

(К) (Г)

Решение: Согласно данному условию sina + sinb = –a и cosa + cosb = -c.

Þ

Þ Þ

Следовательно, (D) — правильный ответ.

29: Если sina, sinb и cosa находятся в G.P, то всегда корни уравнения x 2 + 2x cot b + 1 = 0.

(A) равно (B) реально

(C) мнимая (D) больше 1

Решение: sina, sinb, cosa находятся в G.С.

Þ sin 2 b = sina cosaÞ cos2b = 1 — sin2b ³ 0

Теперь дискриминант данного уравнения равен

4cot 2 b — 4 = 4 cos2b × cosec 2 b³ 0 Þ Корни всегда реальны.

Следовательно, (B) — правильный ответ.

30: Если тогда S равно

(А) (В)

(К) (Г)

Решение:

=

==

Следовательно, (C) — правильный ответ.

31: Если в DABC ÐC = 90 °, то максимальное значение sin A sin B равно

(А) (В) 1

(C) 2 (D) Нет

Решение: sinA sinB =

== =

фунтов стерлингов

Þ Максимальное значение sinA sinB =

Следовательно, (A) — правильный ответ.

32: Если в DABC sin 2 A + sin 2 B + sin 2 C = 2, то треугольник всегда равен

(A) равнобедренный треугольник (B) прямоугольный

(C) остроугольный (D) тупоугольный

Решение: sin 2 A + sin 2 B + sin 2 C = 2 Þ 2 cos A cos B cos C = 0

Þ либо A = 90 o , либо B = 90 o , либо C = 90 o

Следовательно, (B) — правильный ответ.

33. Максимальное значение выражения 2sinx + 4cosx + 3 равно

.

(А) 2 + 3 (В) 2-3

(C) + 3 (D) ни один из этих

Решение: максимальное значение 2sinx + 4cosx = 2.

Следовательно, максимальное значение 2sinx + 4cosx +3 равно

Следовательно, (A) — правильный ответ.

34: Если sinq = 3sin (q + 2a), то значение tan (q + a) + 2tana равно

.

(А) 3 (В) 2

(К) 1 (Д) 0

Решение: дан sin q = 3sin (q + 2a)

Þ грех (д + а-а) = 3 син (д + а + а)

Þ sin (q + a) cosa — cos (q + a) sina

= 3sin (q + a) cosa + 3cos (q + a) sina

Þ –2sin (q + a) cosa = 4cos (q + a) sina

Þ

Þ загар (д + а) + 2тана = 0

Следовательно (D) — правильный ответ.

35: Если cos q =, то одно из значений tan —

(A) желто-коричневая кроватка (B) желто-коричневая кроватка

(C) грех грех (D) ни один из этих

Решение: tan 2 = =

=

= =

= коричневый 2 детская кроватка 2 .

\ tan = ± желто-коричневая кроватка.

Следовательно, (A) — правильный ответ.

36. Если загар 2q. tan q = 1, тогда q равно

(А) (В)

(C) (D) Ничего из этого.

Раствор: tan 2q. загар q = 1

.

Следовательно, (B) — правильный ответ.

37. Если a является корнем из 25, то sin 2a равен

.

(А) (В)

(К) (Г)

Решение: Поскольку, a является корнем

.

Следовательно, (B) — правильный ответ.

38. Уравнение k имеет решение, если

(А) k> 6 (В)

(C) k> 2 (D) Ничего из этого.

Решение: У нас есть k

Но, следовательно,

Сейчас,

Следовательно, (B) — правильный ответ.

39. Общее решение уравнения tan 3x = tan 5x:

(А) x = np / 2, n Î Z (B) x = np, n Z

(C) x = (2n + 1) p, n Î Z (D) Ничего из этого.

Решение: загар 3x = загар 5x

, если n нечетное, то x = np / 2 дает посторонние решения.Таким образом, решение данного уравнения будет иметь вид x = np / 2, где n даже, скажем, n = 2 m, m Î Z. Следовательно, требуемое решение будет x = m p, m Z.

Следовательно, (B) — правильный ответ.

40. Уравнение разрешимо, если

(А) (В)

(C) (D) Ничего из этого.

Решение: У нас

где

для того, чтобы быть реальным.

Дискриминант. . . (1)

Но, следовательно,

. . . (2)

Из (1) и (2),.

Следовательно, (B) — правильный ответ.

41. Набор значений x, для которых равен

(A) f (B) p / 4

(К) (Г)

Решение:

, но это значение не удовлетворяет данному уравнению и сводится к неопределенному виду.

Следовательно, (A) — правильный ответ.

42. Если, то q равно

(А) п / 3 (Б) 2п / 3

(В) п / 6 (Д) 5п / 8

Решение:

или

.

Следовательно, (C) — правильный ответ.

43. Значение выражения —

.

(А) 1/2 (В) 1

(C) 2 (D) Ничего из этого.

Решение: Задано выражение

.

Следовательно, (B) — правильный ответ.

44. Если, то q (только главное значение) равно

(А) п / 3 (Б) 2п / 3

(В) 4п / 3 (Д) 5п / 3

Решение:.

Следовательно, (A) — правильный ответ.

45.Количество решений в интервале [0, 2p] —

(А) 2 (В) 4

(C) 0 (D) Ничего из этого.

Решение:

,

но

\ Решение не существует.

Следовательно, (C) — правильный ответ.

46. Если, то общее решение для q —

.

(А) (В)

(C) (D) Ничего из этого.

Решение:

.

Следовательно, (B) — правильный ответ.

47.Количество решений 11 sin x = x равно

(А) 4 (В) 6

(C) 8 (D) Ничего из этого.

Решение: 11 sin x = x. . . (1)

При замене n на — получаем 11 sin (–x) = –x

Итак, для каждого положительного решения у нас также есть отрицательное решение, и x = 0 удовлетворяет (1), поэтому количество решений всегда будет нечетным.Следовательно, (d0 является подходящим выбором.

Следовательно (D) — правильный ответ.

48. Если, то x равно

(А) (В)

(C) (D) Ничего из этого.

Решение: L.H.S.

и равенство сохраняется для

и Р.H.S.

равенство стариков, если.

Таким образом, L.H.S. = R.H.S. только для.

Следовательно, (B) — правильный ответ.

49. Общее решение для q if, равно

(А) (В)

(C) (D) Ничего из этого.

Решение:.. . (1)

и

(1) может выполняться тогда и только тогда, когда оба равны 1 одновременно. Первое общее значение q, для которого

и

и с периодичностью p

и периодичность 2p, следовательно, периодичность 2p. Поэтому общее решение есть.

Следовательно, (A) — правильный ответ.

50. Если tan a и tan b являются корнями, то значение tan (a + b) равно

.

(А) (В) 1

(C) (D) Ничего из этого.

Решение: корни

и.

.

Следовательно, (C) — правильный ответ.

51. Число решений уравнения tan x = sec x = 2 cos x, лежащих в интервале [0, 2p], равно

.

(А) 0 (В) 1

(К) 2 (Г) 3

Решение: Данное уравнение можно записать как

или –1

Следовательно, необходимое количество решений — 2.

Следовательно, (C) — правильный ответ.

52. Если tan mq + cot n q = 0, то общее значение q равно

.

(А) (В)

(К) (Г)

Решение: Данное уравнение можно записать как

или

.

Следовательно, (A) — правильный ответ.

53. Общее решение уравнения —

(А) (В)

(К) (Г)

Решение: Пусть

или

и

или

Используя их в данном уравнении, мы получаем

или

или.

Следовательно (D) — правильный ответ.

54. Одно из решений уравнения —

(А) (В)

(C) (D) Ничего из этого.

Решение: Данное уравнение можно записать как

или

или

Либо sin q = 0, что дает q = n p

или что дает

Сейчас,

снова

Таким образом, одним решением данного уравнения является

.

Следовательно, (A) — правильный ответ.

55. Решите относительно x и y уравнения:

xy + 3x уютно. у = 14

ху + 3х. y siny = 13

(A) y = где 2n

(B) y = где 2n +

(C) оба

(D) Ни один из этих

Решение: Очевидно, что разделив уравнения x 0, мы получим

по componendo и dividenodo, получаем

или, = 27 =

или, =

разделив числитель и знаменатель на cosy, получим

или,.

siny =, cosy = (когда y находится в 1-м квадранте)

and siny = — и cosy = — (когда y находится в 3-м квадранте)

Когда y находится в первом квадранте.

Когда y находится в 3-м квадранте.

Следовательно, y = где 2n

и y = где 2n +

56.Решение sinx + cosx =:

(А) 2np + (В) 2np —

(C) (D) Ни один из этих

Решение: дано, cosx + sinx =

Þcos x + sinx =

Þ cos

Þ

Þ х = 2np ±.

Þ x = 2np +, 2np — где n Î I.

Следовательно (A, B) — правильный ответ.

57. Решение уравнения tan q. tan 2q = 1 это:

(А) НП + (В) НП —

(К) (Д) нп ±

Решение: Учитывая tan q.загар 2q = 1 Þ = 1

Þ 2 tan 2 q = 1 –tan 2 qÞ 3 tan 2 q = 1

Þ загар q = Þq = np ±

Следовательно (D) — правильный ответ.

58. Найдите общее решение уравнения

sin x — 3 sin 2x + sin 3x = cos x — 3 cos 2x + cos 3x:

(А) (Б) НП —

(К) (Д) нп ±

Решение: дано sin x — 3 sin 2x + sin 3x = cos x –3 cos 2x + cos 3x

Þ 2 sin 2x cos x — 3 sin 2x = 2 cos x cos 2x — 3 cos 2x

Þ sin 2x (2 cos x –3) = cos 2x (2 cos x –3) Þ sin 2x = cos 2x

(cos x 3/2)

Þ загар 2x = 1 Þ 2x = np + Þ x =, n I.

Следовательно, (C) — правильный ответ.

59. Решите относительно x уравнение sin 3 x + sin x cos x + cos 3 x = 1:

(А) 2 МП (В) (4n + 1)

(C) Оба (D) Ни один из этих

Решение: Данное уравнение имеет вид sin 3 x + cos 3 x + sin x cos x = 1

Þ (sin x + cos x) (sin 2 x — sin x cos x + cos 2 x) + sin x cos x — 1 = 0

Þ (1 — sin x cos x) [sin x + cos x — 1] = 0

Либо 1 — sin x cos x = 0 Þ sin 2 x = 2, что невозможно

Или, sin x + cos x — 1 = 0 Þ cos (x — p / 4) = Þ ±

Þ x = 2mp и x = (4n + 1)

Следовательно, (C) — правильный ответ.

60. Уравнение e sinx — e –sinx — 4 = 0 имеет:

(A) нет реального решения (B) одно реальное решение

(C) два реальных решения (D) не могут быть определены

Решение: Данное уравнение можно записать как

e 2 sin x — 4e sin x — 1 = 0 Þ e sin x = = 2 +

Þ sin x = ln (2 +) (ln (2 -) не определено как (2 -) отрицательно)

Теперь, 2 +> e Þ ln (2 +)> 1 Þ sin x> 1

Что невозможно.Следовательно, нет реального решения.

Следовательно, (A) — правильный ответ.

61. Если tan (p cos x) = cot (p sin x), то

(А) (В)

(C) 0 (D) Ничего из этого.

Решение: Учитывая, что tan (p cos x) = cos (p sin x)

или

.

Следовательно, (B) — правильный ответ.

Чтобы узнать больше, купите учебные материалы по Тригонометрия , включая учебные заметки, заметки о пересмотре, видеолекции, решенные вопросы за предыдущий год и т. Д. Также дополнительные учебные материалы по математике можно найти здесь .

Линейные неоднородные дифференциальные уравнения второго порядка с постоянными коэффициентами

Пример 2.{\ prime \ prime} _1} = 0. \]

Подстановка этого в дифференциальное уравнение дает:

\ [ {0 + A — 6 \ left ({Ax + B} \ right) = 36x, \; \;} \ Rightarrow {A — 6Ax — 6B = 36x.} \]

Последнее уравнение должно быть действительным для всех значений \ (x, \), поэтому коэффициенты с одинаковыми степенями \ (x \) в правой и левой частях должны быть идентичны:

\ [\ left \ {\ begin {array} {l} — 6А = 36 \\ А — 6В = 0 \ end {array} \ right .. \]

Из этой системы находим, что \ (A = -6, \) \ (B = -1.{\ prime \ prime} _1} = — 4A \ cos 2x} — {4B \ sin 2x.} \]

Подставляя это обратно в дифференциальное уравнение, получаем:

\ [ {- 4A \ cos 2x — 4B \ sin 2x} + {16 \ left ({A \ cos 2x + B \ sin 2x + C} \ right)} = {\ cos 2x + 1,} \]

\ [ {- 4A \ cos 2x — 4B \ sin 2x} + {16A \ cos 2x + 16B \ sin 2x + 16C} = {\ cos 2x + 1,} \]

\ [{12A \ cos 2x + 12B \ sin 2x} + {16C} = {\ cos 2x + 1.} \]

Последнее выражение идентично. Поэтому мы можем написать следующую систему уравнений для определения коэффициентов \ (A, B, C: \)

\ [ {\ left \ {\ begin {array} {l} 12А = 1 \\ 12В = 0 \\ 16C = 1 \ end {array} \ right.